Sie sind auf Seite 1von 70

Special Report

Spandrel Beam Behavior


and Design

Charles H. Raths
Senior Principal
Raths, Raths & Johnson, Inc.
Structural Engineers
Willowbrook, Illinois

Presents common precast spandrel beam distress


causes, discusses types of loads applied to spandrel
beams and overall torsion equilibrium requirements,
provides design relationships for spandrel beams,
offers design criteria for spandrel beam connections,
sets forth basic good design practices for spandrel
beams, and gives design examples.

62
CONTENTS
Introduction.............................................64

Pastand Current Problems ... ............................. 64


Types of Problems Actual Problems

Typesof Applied Loads ................................... 70


Gravity Loads—General
Beam Loading Spandrel Ledges
Horizontal Loads Volume Change Forces
Beam End Connections Frame Moment Forces

General Design Requirements ............................. 79


Internal Torsion and Shear
Beam End Torsion Web Flexure
Ledge Attachment Corbel End Behavior
Ledge Load Transfer Beam Flexure

Considerations for Connnection Design ..................... 92


Connection Systems Frame Connections
Connection Materials Reinforcement Considerations
Connection Interfacing Column Influence

Good Design Practice ...... .............................114


Cross-Sectional Dimensions Connections
Reinforcement Bearing Considerations
Tolerances and Clearances Ultimate Load Factors
Corrosion Protection Supporting Columns
Loads Inspections

ClosingComments .......................................120

Acknowledgments........................................120

References..............................................121

Notation................................................122

Appendix—Design Examples ..............................124

PCI JCURNALJMarch-April 1984 63


S pandrel beams, reinforced or pre-
stressed, are important and func-
tional elements of precast concrete
substantial repair costs, construction
delays, temporary loss of facility use,
and various legal entanglements.
structures. Given the current design
knowledge presented by the PCI Design Types of Problems
Handbook , and the PCI Connections
The types of problems experienced by
Manual,' and basic fundamentals of
spandrel beams can be categorized as
structural engineering mechanics, it
follows:
would seem the topic of spandrel beam
• Overall torsional equilibrium of
design does not merit further review.
the spandrel beam as a whole.
Yet, considering the number of precast
• Internal torsion resulting from
framed structures experiencing various
beams not being loaded directly
types of problems and distress with
through their shear center.
spandrel beams, something is amiss re-
• Member end connection.
garding the designer's understanding of
• Capability of the spandrel beam's
spandrel beam behavior and design re-
ledge and web to support vertical
quirements.
loads.
The purpose of this paper is to review
• Volume change restraint forces in-
some of the common problems typically
duced into spandrel beams.
associated with spandrel beams, and to
suggest design requirements for them.
Towards this end, discussions herein Actual Problems
are devoted to actual problems experi- The fallowing discussion of actual
enced by spandrel beams, load sup- case histories relates to projects where
porting functions, general design re- photographs can be used to illustrate the
quirements, connections, and good de- problems associated with the above
sign practice. While spandrel beams are listed items. This is not meant to imply
used to satisfy a variety of structural that aspects not covered by the ease
functions, this presentation will em- histories are less significant than those
phasize simple span load supporting considered.
members. The conditions associated with lack of
overall beam torsion equilibrium are
shown in Fig. 1. The non-alignment of
PAST AND CURRENT applied loads and beam end reactions
PROBLEMS can be seen in Fig. Ia. Fig. lb shows the
beam rotation crushing of the topping
Difficulties related to spandrel beams concrete caused by the lack of beam and
occur for members supporting floor torsional equilibrium connections. In
loads, those which are part of a moment Fig. 1, the torsional rotation problem is
frame, or members that are neither compounded by the presence of neo-
gravity supporting nor part of a moment prene type bearing pads at the beam and
resisting frame. Typically, simple span tee bearings where the pad deformation
spandrel beams as utilized in parking causes further torsional roll. Generally,
garages appear to be the most problem the problem of overall spandrel beam
prone although similar troubles develop torsional equilibrium represents the
in office or other "building" type struc- majority of the difficulties experienced
tures. by spandrel beams.
The difficulties and problems dis- Internal spandrel beam torsion dis-
cussed usually do not result in the col- tress, frequently within distance "d" of
lapse of spandrel beam members. the spandrel beam's ends, is often ob-
Nevertheless, these problems can create served.* Fig. 2a illustrates a spandrel

64
Fig. 1 a. Lack of overall torsion equilibrium: Fig. lb. Lack of overall torsion equilibrium:
underside view showing non-alignment of top view showing beam rotation and
applied loads and end reaction. crushing of topping (arrows).

beam loaded eccentric to its shear The lack of overall spandrel beam tor-
center and shows the beam's torsional sion equilibrium is often reflected at
equilibrium connections (note the tee column support connections resulting in
legs on either side of the column result high nonuniform bearing stresses
in a concentrated load within distance caused by spandrel beam torsion roll.
"d"). Internal torsion cracks in the Fig. 3a shows the applied tee loads not
spandrel beam web can be observed in aligning with the support column reac-
Fig. 2b. These torsion cracks result from tion. Lack of required overall beam tor-
a combination of internal beam torsion sion equilibrium connections during
and the flexural behavior influence of erection often results in excessively
the beam end torsion equilibrium con- high (usually localized) bearing distress
nections. Torsion distress (cracking) at as reflected by Fig. 3h. A variation of the
beam ends is a common problem af- same problems caused by spandrel
fecting spandrel beams when designs do beam torsion roll (lack of beam end tor-
not consider the influence of loads sion equilibrium connections) is pre-
within distance "d" or torsion equilib- sented by Fig. 4a which produced col-
rium connection forces. umn corbel failures. Fig. 4h indicates
the magnitudes of torsion roll that can
develop bearing distress when the
'Note: "d" denotes the distarive from the extreme
compression fiber of the member to the centroid of necessary torsion equilibrium connec-
flexural tension reinforcement. tions are not provided.

PCI JOURNAL/March-April 1984 65


Fig. 2a. Underside view of beam end torsion cracking within distance "d"
caused by equilibrium connections and internal torsion (arrow indicates location
of reaction).

Fig. 2b. Top view of end cracks (arrows).

66
N

/ çc

•C
.3
TOP
Fig. 3a. Underside view of bea •ing distress
resulting from torsional roll duo to lack of
erection connections (left arrow points to
beam reaction and right arrow indicates
applied loads). Fig. 3b. Close-up of typical bearing distress.

Fig. 4a (left). Upward view of column


corbel failure caused by lack of any overall
torsion equilibrium connections.

Fig. 4b (above). Example of torsional roll


magnitude at a non-distressed corbel.

PCI JOURNAL/March-April 1984 67


Fig. 5. Beam ledge end corbel failure
(applied loads and reaction in alignment).

An example of distress resulting from


inadequate considerations of structural
behavior, even though overall torsion
equilibrium is assured by applied loads
and beam reactions aligning, is given by
Fig. 5. The spandrel beam haunch, at
the beam end, acts like a corbel, and Fig. 6a. Overall view of torsion equilibrium
when not reinforced to resist the entire tension insert connection (top arrow
beam reaction, as in Fig. 5, distress or indicates bearing pad location and lower
failure develops. arrow points to failed tension insert).

Fig. 6b. Close-up view of failure (left arrow points to insert and right arrow indicates shear
cone crack failure plane).

68
[1
Figs. 6a and 6b show yet another vari-
ation of spandrel distress resulting from
a failure of the overall torsion equilib-
rium tension insert connection. The
non-alignment of the beam end reaction
and the applied tee loads causing tor-
sion were resisted by a horizontal couple
developed by a steel bolted insert ten-
sion connection near the top ofthe ledge
and bearing of the beam web against the
column top.
The dap behavior of spandrel beam
haunches (or ledges), particularly at
beam ends, sometimes is neglected re-
sulting in haunch failures as shown in
Fig. 7. This particular member (Fig. 7)
failed because the necessary reinforce-
ment was not present at the beam end,
and because the concrete acting as plain
concrete did not have the necessary
capacity to resist applied shear and flex-
ure forces, Another category of span-
drel dap and internal torsion problems is
shown by Fig. 8, where insufficient dap Fig. 7. Ledge failure near beam end.
reinforcement, AIA , was used (refer to
Fig. 40 and the Notation for the full
meaning ofA,h).
Spandrel beams employing relatively
thin webs have, depending on the web
thickness and the type of web rein-
forcement (shear and torsion), suffered
separation of the entire ledge from the
beam as illustrated by Fig. 9. This type
of distress results from inadequate con-
siderations of how tee reaction loads to
the ledge are transmitted into the beam
web. The ledge transfer mechanism re-
sembles a dap (an upside down dap)
subject to both flexure and direct
tension.
The actual case histories reviewed, and
many others not commented upon, indi-
cate that some designers do not under-
stand the entire behavior of spandrel
beams. The problems related to span-
drel behavior occur in all types of
structures, are not confined to any one Fig. 8. End support dap distress caused by
geographic area, and inevitably result insufficient dap and torsion reinforcement
when one or more of the basic en- (upper arrow indicates epoxied dap cracks
gineering fundamentals are missed or and lower arrow points to modified end
ignored. support).

PCI JOURNAUMarcn-April 1984 69


Fig. 9. Complete separation failure of ledge from beam web.

TYPES OF APPLIED LOADS internal torsion, respectively, caused by


gravity loads eccentric to the beam's
Spandrel beams are subjected to a va- shear center.
riety of loads. These loads result from Fig. 10 shows the elements which are
applied gravity forces, horizontal impact fundamental to the load behavior re-
forces (e.g., parking garage spandrels), sponse of spandrel beams. Relative to
end connections, ledges transmitting Fig. 10a, ife e equals or exceeds e, (end
loads to the spandrel beam, volume reactions align with applied loads when
change forces, and frame moments. e, = e2 ), then no connections are re-
These loading cases can act separately quired to develop the resisting torque
or in combination. The discussions T,, necessary for overall gravity torsion
herein relate to precast spandrel beams equilibrium. The influence of loads
acting as simple span load supporting within distance "d" for spandrel beams
members. of significant height is shown in Figs.
10b and 10c. ACI 318-77 3 Sections
Gravity Loads — General Beam
11.1.3.1 and 11.1.3.2 indicate that the
Loading design for shear need only consider
The gravity loads of a precast spandrel shears at "d" or "d/2" while Section
beam are typically concentrated and re- 11.6.4 indicates that torsion, for non-
sult principally from its tee legs as prestressed members, need only he con-
shown in Fig. 10. Fig. I(1a presents the sidered at distance "d" or beyond. The
overall equilibrium requirements for ACI 318-77 Code, relative to spandrel
concentrated loads applied to a simple beam design, is incorrect (though not
span spandrel beam. Figs. lob and 10c the Commentary) since using the Code
show the resulting beam shear and procedures can result in precast span-

70
P6 TB
P (NOT SHOWN)
J

er Pq
RB
P6

P2

j IP1
SHEAR
CENTER
ALL LOADS EQUAL AND
T AUNIFORMALLY SPACED

RA

(a) Loads

1 UNIFORM LOAD = LP
` 2 d,

RA 3

d. RB
4

6 \

(b) Applied Shear

UNIFORM LOAD = SPe-l-


2

Tp=RAe 1 3

TB =Rse,

(c) Applied Torsion

Fig. 10. General gravity loads showing applied shear and torsion diagrams.

PCI JOURNAL/March-April 1984 71


SPANDREL
BEAM

CURB
(OPTIONAL)

TOPPING

TYPICAL
TEE STEM
TEE
END REACTION FLANGE
AT COLUMN

(a)

H (10K WORKING LOAD)

H,>H2
hH

Ht
SHEAR
CENTER

H2

HORIZONTAL BEAM SPANNING


COLUMN TO COLUMN

(b)

Fig. 11. Horizontal loads acting on spandrel beam.

72
drel members being designed for only acting upon the beam, not just those lo-
two-thirds of the applied shear and tor- cated "d/2" or "d" beyond the end sup-
sion load in the end regions, and the port reaction. Typically, the top H force
concentrated influence of torsion of the couple is developed by flexural
equilibrium connections at the beam behavior of the beam web. The combi-
end being neglected. nation of the torsion equilibrium web
flexural stresses, shear stresses, and
internal torsion stresses results in the
Horizontal Loads
45-deg cracking repeatedly observed in
Spandrel beams, as used in parking spandrel beams as illustrated in Fig. 12.
garages, can he required to restrain au- Also, the 45-deg cracking is affected by
tomobiles which result in horizontal ledge concentrated P loads located near
impact loads as demonstrated in Fig. 11. the beam's end inducing additional web
Horizontal loads to spandrel beams can stresses.
be applied at any location along the A different force pattern results when
beam's span where "a" of Fig. ha can the spandrel end reaction acts upon the
vary from zero to L. Both load support- ledge. Overall torsion equilibrium ofthe
ing spandrels and non-load supporting spandrel beam is achieved by the reac-
spandrels can be subjected to these tion force R aligning with the applied
horizontal loads.
Fig. 11b illustrates the "cantilever'
method of resisting horizontal loads
which requires the floor diaphragm
(force H,) and the bearn's haunch acting
as a horizontal beam (force HQ ) to be the
load resisting elements. Alternately,
horizontal loads can be resisted by the
spandrel beam acting as a simple span
between its end supports providing that
torsional equilibrium is maintained by
the required number of connections, or
by the beam having adequate torsional
strength to transmit the torsion loading
to the end torsion equilibrium connec-
tions.

Beam End Connections


Spandrel beam end connections
which induce forces into the beam can,
for the sake of simplicity, be divided
into three types. The first type is that
associated with the beam's overall tor-
sional equilibrium, the second type
deals with "corbel" support behavior,
and the third pertains to dapped end
support,
The H forces applied by overall beam
torsion equilibrium end connections are
shown in Fig. 12. The magnitude of the
horizontal force couple H providing
equilibrium results from all the loads

PCI JOURNAUMarch-April 1984 ^3


ledge concentrated loads P (neglecting
the beam weight), or being beyond the
load P. The projecting beam ledge acts
like an "upside down" corbel, as shown
in Fig. 13, and the projecting ledge must
be treated as a corbel if the applied end
forces are to be properly considered.
The ends of spandrel beams are
sometimes dapped. When daps exist,
and the applied concentrated ledge
loads P do not align with the reaction R,
the forces and stresses resulting from
the combined action of the dap and the
equilibrium forces (Fig. 12) require
complete understanding by the de-
signer. Fig. 14 illustrates the combined
action of all forces when a spandrel end
support dap is present and the cracking
which can develop.
End connection forces applied to
simple span spandrel beams also can re-
sult from forces necessary to achieve
column equilibrium and concrete vol-
ume change deformations. However,
these factors are more appropriately dis-
cussed elsewhere in this paper.

Spandrel Ledges
The ledge, or haunch, of a spandrel
beam is the usual mechanism for trans-
fer of the applied concentrated loads to
the beam web where the web in turn punching shear which also could be
transmits these concentrated loads to considered as "upside down" dap shear.
the spandrel support reaction. The The location of a concentrated load
Iedge transfers load to the beam web via along the beam's ledge influences the
flexure, direct shear, punching shear, ledge's ability to transmit load gener-
and web direct tension. ated forces. Fig. 16 illustrates the
Two flexure behavior paths exist for punching shear transfer of ledge loads to
transfer of ledge loads to the spandrel the web for loads applied near the span-
web. As shown in Fig. 15, one path is at drel's end and away from the end.
the vertical interface of the ledge and
beam web while the other is at a hori- Volume Change Forces
zontal plane through the spandrel web Volume change forces resulting from
in line with the top surface of the ledge. restraint of concrete deformations
The forces P and N not only induce flex- caused by shrinkage, creep, and tem-
ure but also create a state of direct ten- perature can occur at the beam's sup-
sion. Both force paths must he ac- porting ledge, all connections, and the
counted for. beam end bearing support. The volume
The dominant shear transfer mode of change forces can be axial or rotational
ledge loads to the beam web is by as reflected in Fig. 17.

74
R H
30 TO 45 CRACK

R
Fig. 14. Combined action of dap and torsion equilibrium forces.

COMBINED WEB FLEXURE P


AND DIRECT TENSION

LEDGE FLEXURE AND


DIRECT TENSION

Fig. 15. Ledge to beam flexural and tension paths.

PCI JOURNAL/March -April 1984 75


Axial volume change forces producing of the frame's stiffness center position
tension in members generally exist, and (position where horizontal volume
their magnitude depends on the rigidity change movements are zero).
of restraint to the volume change Sun induced temperature differences
movements. For example, welded con- between the top and bottom of a span-
nections or hard high friction connec- drel beam, where the top surface tem-
tions can develop large N. forces perature is greater than the bottom, can
whereas members joining one another create a positive end moment if the
through bearing pads (soft connections) beam's end connections are actually
can result in only minimal N. forces. rigid. The positive end moment is de-
The N. forces acting on spandrel beams veloped by a combination of horizontal
can control connection designs, and de- and vertical force couples as shown in
pending on their magnitude can mate- Fig. 17b. However, the forces of the
rially reduce the beam's shear and tor- couple typically can be neglected for
sion strength if the N. force acts parallel spandrels since in reality they are small
to the beam's length. Another factor in- because: minute deformations of the
fluencing axial volume change forces in connections themselves relieve the ro-
a spandrel is the location of the beam tational restraint; restraint to expansion
within the building frame horizontally reduces the horizontal couple force; re-
and vertically in addition to the location straint of shrinkage and creep contrac-
TOP
CONNECTIONS

N CONNECTIONS TO N
SUPPORTED MEMBERS

APPLIED LOADS
TO LEDGE

BEAM REACTION
BEARING
N (a} Axial Forces

RIGID
CONNECTIONS
T2

T1

HT
I TEMPERATURE
EXPANSION
T2>T1

NT
RESOLVES INTO HORIZONTAL
RESISTING AND VERTICAL FORCE COUPLES
ROTATIONAL (SEE NON-SHADED ARROWS)
MOVEMENT

NT
NT RESULTS FROM
EXPANSION RESTRAINT HT
VT V1

(b) Rotational Forces

Fig. 17. Types of applied volume change forces.

PCI JOURNALJMarch-April 1984 77


,1 .^_ H M

M(_
OHM

(a) (b)
Fig. 18. Moment frame connections.

tion produces rotations opposite to the of a rigid frame although this is not a
temperature end rotations; and, for nar- common application. When spandrels
row width beam tops, the internal top to are part of a moment frame, connections
bottom temperature variations are small at the beam's end are used to develop
resulting in very minor beam axial and horizontal couple forces as shown in
rotational deformations. Fig. 18. The horizontal couple forces
Normally, temperature camber influ- can be of varying magnitude and direc-
ence is of greatest concern to members tion depending upon the mode of the
having a wide top flange width com- frame's sway, deformations, and type of
pared to its bottom flange such as dou- lateral load applied. The moment frame
ble or single tee type beam cross sec- horizontal couple forces are additive or
tions when rigid end connections exist. subtractive to the other forces at the
Typically (in northern climates), sun in- spandrel's end(s) when determining
duced temperatures result in the flange overall design forces. If the moment
average temperature being 30 deg F (17 frame spandrel beam also supports
deg C) greater than the web tempera- gravity loads, the gravity loads produce
ture. additional moments which must be
combined with other frame moments.
Frame Moment Forces Gravity dead loads may or may not cause
end moments, depending upon when
Spandrel beams can serve as members frame connections are made.

78
WES

T i
HU

'ep eH
SHEAR
CENTER PU

eN
*
i Nu

TORQUE = Puep- HueH - NueN


NOTE:
LOADS PU. HUAND NUDO NOT ALWAYS ACT SIMULTANEOUSLY
DESIGN FOR CONTROLLING CASE

Fig. 19. Torque forces.

GENERAL DESIGN Internal Torsion and Shear

REQUIREMENTS Spandrel beam torsion results when


applied horizontal and vertical loads do
Spandrel beam design requires con- not pass through the beam's shear cen-
sideration of how all the various applied ter. The resulting torsion to the beam, at
loadings are transmitted from their point any cross section, is the sum of the
of application to the beam and then sub- torques (shear force times distance from
sequently to the structural element(s) the shear center) acting at that cross
supporting the spandrel itself. Basic de- section. Moreover, it is possible that the
sign requirements, exclusive of the con- loads acting on the spandrel can vary
nections to columns or other supporting from the time of erection to when all
structural elements, are: time-dependent volume change loads
• Internal torsion and shear act. Each loading case requires evalua-
• Beam end torsion tion to determine which controls the de-
• Ledge attachment to the web sign.
• Ledge load transfer Fig. 19 shows the loads applied to a
• Web flexure resulting from torsion spandrel beam eccentric to the shear
equilibrium center. The determination of the shear
• Ledge acting as a corbel at beam center is provided by Fig. 20 4 for a ho-
end reaction mogeneous uncracked section. Once the
• Beam flexure applied torsion and shear are known, the

PCI JOURNAL./March-April 1984 79


internal torsion and shear reinforcement force is in lbs or kips, the stress is in psi
can he determined following ACT 3183 or ksi, and = 0.85.
requirements for reinforced members or Note that equal to 0.85 results from
employing other relationships 9' for pre- using 0 = 0.90 for flexure and j„ = 0.94
stressed members. If only vertical loads such that the denominator for Eq. (1) is:
are applied to the spandrel, the shear
center can quite conservatively be as- Oi ti d^f, = 0.9 (0.94) df, = 0.85 d^f^,
sumed to align with the beam's web
vertical centerline. The spacing of the A,, reinforcement,
horizontally and vertically, should not
be greater than 12 in. (305 mm).
Beam End Torsion
If the amounts of the vertical stirrups
Beam end torsion is defined as the and horizontal bars distributed about
torsion at the beam's end, within the the spandrel's perimeter differ, the
distance "d" or "d/2," resulting from the quantity ofA, L, available depends on the
torsion equilibrium end connections. lesser amount of A, rD and A" (see Fig.
Typically, beam end torsion created by 21b):
top and bottom connections is charac-
terized by a single crack, inclined at A. = 2 (1 Al )
(2)
about 45 deg, having a nominal width of V2
0.015 in. (0,38 mm) or greater. The 45-
deg crack results from flexure in the or
spandrel's web induced by the beam top
A 2C A) (3)
Ha connection force (see Fig. 21a) re-
quired to maintain overall torsion equi- ^2
librium. The magnitude of the H. forces whichever results in the smaller value
is that required to resist all loads eccen- forA^.. The I A., or!. A,,., represents the
tric to the shear center, and not just total amount of reinforcing steel within
those loads at and beyond "d" or the effective dimensions defined by Fig.
from the beam's vertical end support. 21c. Anchorage of the A. and A 1 rein-
The end of the beam length required to forcing bars is important. Both ends of
transfer the H. equilibrium forces to in- the, steel must be anchored by hooks
ternal torsion behavior can be consid- or closed stirrups. The A,, bars require
ered to be within the distance "d" from hooking at the end of the beam or a "U"
the beam's end. shape for proper anchorage.
Beam end reinforcement to resist the The vertical and horizontal rein-
web flexure caused by the H. forces can forcement comprising the A steel at the
be supplied by horizontal and vertical beam's end is in addition to any other
bars. The orthogonal bars consist of steel reinforcement necessary to resist shear
areas A,, A., and A, (according to ACI since the A, and shear reinforcement
318-77) located on the ]edge side of the share a common crack plane.
beam's web (see Fig. 21b). The end Another type of special beam end tor-
reinforcement effective in providing the sion is discussed in a following section
necessary 45-deg A u steel of Fig. 21d is titled "Corbel End Behavior." This spe-
given by Fig. 21c. The web A,, rein- cial case, as for the above, requires the
forcing steel required for ultimate beam's end to be designed to resist the
strength is: total torsion acting at the beam's sup-
port.
A. = H€',(1)
04J, Ledge Attachment
where all dimensions are in inches, the Attachment of the spandrel beam

80
'.EG 1 - RECTANGLE w 1 by

EG 2-RECTANGLE W 2 h2

'DINT A - INTERSECTION OF VERTICAL &


HORIZONTAL LEG CENTERLINES
( Y-Y & x-x )

'DINT B - SHEAR CENTER

DOTE: IF W, AND W 2 SMALL. ex .ey -O.


AND SHEAR CENTER B IS AT A.
CONSERVATIVE TO SELECT SHEAR
CENTER AT POINT A.

CALCULATION OF ex

h2 I2
I__
J+J2) h2 h2w/12
eX 2 ( 2 +h2w/i2J

h2 h2w2'
ex= 2 [Wlh,3+h2w2]

CALCULATION OF ey

h1 I^ h1 h1wj /12
ey 7T
+12) ! 2 [hiw13 /12+w2h2/12

h 1h1w 1a
ey
a 2 h l w l' + w2hz

Fig. 20. Calculation of spandrel beam shear center.

PCI JOURNAL/March-April 1984 $1


U 45 CRACK H - 45C

N, Hu

(a) End Web Cracking

H U CENTROID cAwv^2 +At)

t
A Hu
Awt -L 2
W
d ww e

e
#/
Hu

(b) End Torsion Nomenclature (d) End Aw

TYPICAL A1 REINF.

f ^I
HORIZONTAL REINF. EFFECTIVE
OVER THIS DIMENSION

TYPICAL Av + At REINF.
VERTICAL REINF. EFFECTIVE
OVER THIS DIMENSION

(c) Orthogonal End Reinforcement

Fig. 21. Beam end torsion equilibrium reinforcement.

82
ledge to the web can be either by the ultimate where k = 1 for nor-
strength of plain concrete or by rein- mal weight concrete and 0.85 far
forcing steel depending upon the beam sand lightweight concrete
dimensions, concrete strength, and
The ft maximum value at ultimate of
magnitude of the ledge load. The posi-
3a f},' for ff is derived from the tradi-
tion of the load Vu also can have an in-
tional concrete cone punching shear di-
fluence on the ledge to web capacity.
rect tension limit of 4 , f { using a phi
Accordingly, considering the fabrication factor of 0.75 and k to account for the
tolerances of the framing members, in- concrete unit weight. Moreover, consid-
terfacing erection tolerances, and de- ering the nominal reinforcements which
formation characteristics of the ledge usually exist in both the beam web and
supported member, the position of V. ledge, the value of 3X , f is a lower
(see Fig. 22) should be located at ^
hound value when ACT 318-77 or ACI
The ledge to web horizontal attach- 322-72 8 allows f, = q55 f f (0 = 0.65) for
ment is considered similar to the be-
unreinforced concrete flexure at ulti-
havior of two rigid bodies where sep-
mate. If fg exceeds 3A , ,, attachment
aration would occur along the entire reinforcement is required.
length of the beam's web on the attach-
Reinforcement for attachment of the
ment plane of Fig. 22a. Two separate
spandrel ledge to the beam's web is
conditions can exist as shown by Fig.
shown in Fig. 22c, along with the pa-
22a, and concern an end load (near the
rameters for determining the amount of
beam's end) or an inner load (away from
reinforcing steel required. Typically,
the beam's end). The concentrated loads
the depth of the ultimate stress block is
acting are shown in Fig. 22, but the
less than 1 in. (25.4 mm) for usual design
ledge attachment also must consider
conditions. Assuming the stress block
uniform loads. Uniform loads would
centroid is at 0.5 in. (12.7 mm) (a/2) per
produce V.'s equal to the uniform load
Fig. 22c, and summing moments about
times the ledge length being examined.
this centroid, the reinforcement for at-
The ultimate attachment strength of a
taching the ledge to the web is:
non-reinforced ledge depends on the
tensile strength of the concrete. The __ V„ (b,,. + 3/a 1p – '/s)
point of maximum ultimate tension is A (5)
Ae 0 Jv (b.. — dA — 1/2)
(see Fig. 22b). The maximum stress f,
results from a combination of direct ten- where 0 = 0.85, all dimensional units
sion and flexure, and is: are in inches, the force is in lbs or kips,
the stress is in psi or ksi, and the ledge to
f = V,, + 6 V„ (b,o/2 + 3/4 1 p) (4) web reinforcement is uniformly dis-
bum barn tributed over the length m as previously
defined.
where s and de have maximum values in
The Ag reinforcement of Eq. (5) can in
accordance with Fig. 22. All dimen-
part or whole be supplied by web stir-
sional units are in inches and the force is
rups in the beam. The ledge attachment
in lbs or kips. Nate that:
reinforcement usually is not additive to
m = s for inner concentrated loads shear and torsion reinforcing because
m = d,. + sl2 for end concentrated the A, web steel reinforces a different
loads crack plane. The amount of beam web
reinforcing will be controlled by the
m = length selected for analysis of
greater of the requirements for comn-
uniform load and V. = ni times
hined shear and torsion or ledge attach-
unit uniform load
ment. The spacing of A, bars should not
f, = 3. vrT psi tension maximum at exceed 18 in. (457 mm).

PCI JOURNAL.IMarch-April 1984 83


EN° Nu

s
Nu de { 1h

de
h

(a) Punching Shear Transfer

—Ash THIS LEG


ONLY

3/4 L p 3^4 p

VU vu
As
DAP N JV
SHEAR ^` ^' u
PLANE• HAIRPINSAT
VERTICAL
BEAM END
^ SHEAR PLANE

Ah = Ash/2

(b) Dap Shear (c) Ledge Transfer Reinforcing


Fig. 22_ Beam ledge attachment for non-reinforced and reinforced sections.

84
Ledge Load Transfer tests by the author have indicated that
The spandrel beam's ledge transfers the ultimate dap unreinforced concrete
uniform and concentrated loads to the shear stress capacity, when using the
web by shear and flexure. The engi- vertical shear plane as the measure, is
neering procedures presented for 1.2X[J for normal weight and sand
transfer.of the ledge loads are based on lightweight concretes having an f =
the PCI Design Handbook,' with some 5000 psi (34 MPa) and the load applied
variations, and are restated to keep this on the ledge at 3/alp . A value of 3, is
presentation all inclusive for spandrel used on all shear planes in Eq. (6) while
beam design. Eq. (8) uses a value of 2,fJ on the pro-
The ledge load transfer must satisfy jecting ledge shear area hI,,.
concrete punching shear if special shear Applied V. forces in excess of the un-
reinforcement is not to be used. Fig. 23a reinforced V. capacities determined by
portrays concentrated loads applied to Eqs. (6) through (10) require dap rein-
the ledge and the ultimate punching forcement, .A (see Fig. 23c). The neces-
shear pattern. The ultimate V. capacity sary A,, to resist the applied V. is:
for inner ledge loadings and end load-
ings when de equals or exceeds 2h is: A. (11)
v
Fors>b, +2h
where 0 = 0.85, the V. force is in lbs or
V. = 30hx,1l (2lp + b, + h) (6) kips, and the stress f, is in psi or ksi.
The ultimate shearing stress on the
Fors -_b, +2h
ledge vertical shear plane when A,,
V. = 0 shA (7) steel is used (see Fig. 23b) should not
exceed 10 k,f7 . The vertical shear
where 0 = 0.85, all dimension units are plane area for concentrated loads spaced
in inches, ff is in psi, and h is as previ- further apart than b, + 2h is h (b r +h).
ously defined. Fig. 23 can be used as a guide in deter-
The ultimate V. capacity for end ledge mining the vertical shear plane area for
loading, where de is less than 2h, is: other loadings.
The A Rh reinforcement determined
Fors>b, +2h from Eq. (11) should be totally with bt+
h for inner and end loads, d, + (b, + h)/4
V. = 4hx7 [21k + (b, + h)12 + d e ] (8)
for end loads when d, is less than (b t +
For s _- b, + 2h h)/2, and within s for closely spaced
concentrated loads or uniform loads
V„ = 0hx VT, (dd + sf2) (9) unless a greater spacing can be justified
by the longitudinal reinforcement in the
where the notation is the same as for spandrel ledge laterally distributing the
Eqs. (6) and (7).
concentrated load.
As for closely spaced concentrated Longitudinal reinforcement,A h , in the
loads, the ultimate capacity for uniform beam's ledge should he, at a minimum,
loads applied to the ledge is the same as equal to A,Al2 to insure all A,,, reinforce-
given by Eq. (7), which for an s of 1 ft ment over its distribution width is en-
(0.305 m) provides: gaged. The longitudinal reinforcement
V„=th12hAvf,' (10) distributes concentrated ledge loads
along the ledge by both dowel shear and
Eqs. (7) through (10) use an ultimate flexure when the reinforcement is lo-
shear stress of 1,/7 on the vertical shear cated near the ledge top as shown in
plane shown in Fig. 23b. Unpublished Fig. 23c.

PCI JOURNALIMarch-April 1984 B5


Nu

N de42h

ee
h

(a) Punching Shear Transfer

3I
Ash THIS LEG
ONLY

^41p 3I4Lp

V Vu

DAP AS
N N
SHEAR _
PLANE HAIRPINS AT
VERTICAL
SHEAR PLANE REAM END

Ah = Ash/2

(b) Dap Shear (c) Ledge Transfer Reinforcing

Fig. 23. Ledge load transfer.

86
The A h at the end of the beam should members it supports and bottom con-
confine the end of ledge, and typically nections at the spandrel's end vertical
have a hairpin configuration which laps reaction. This condition is illustrated in
to the continuous ledge A. The steel re- Fig, 24 where there is no top end con-
quired by Eq. (11) is not additive to the nection to provide the necessary overall
stirrups required to resist web shear and torsion equilibrium. A similar loading
torsion since it basically reinforces a condition can occur when horizontal
different crack plane. However, when loads are applied to the beam web (see
A,^, is required, more stirrups may be Fig. 11b), except that the forces have a
necessary at the concentrated V„ loca- direction opposite of those resulting
tions so the total steel area provides the from vertical load torsion equilibrium.
calculated A 3h distributed over the Fig. 24a presents a typical cross sec-
lengths previously discussed. If A sh is tion away from the beam ends showing
not provided by additional stirrups, then the w developed by the topping
carefully anchored separate reinforce- acting against the web. The distribution
ment on the ledge side of the web of the unit torsion load w, V against the
should be used. web is given by Fig. 24h for a spandrel
Ledge flexural reinforcement is re- beam having equal tee stem reactions
quired to resist the applied V„ and N. and uniform stem spacing where w,^ is:
loads. The A, steel (see Fig. 23c) can be
selected from: T
iUty (13)
(L/2) h„
)1 (12) and
A,= -/,-CV_Q4d )+N„(
where ¢ = 0.85, forces are in lbs or kips, H = w,„ (L/2) (14)
dimensions are in inches, and the f„
stress is in psi or ksi. Note that T in Eq. (13) represents the
The A, reinforcing steel using the bent total overturning torque for half the
configuration of Fig. 23c does not pro- beam, and the other variables are de-
fined in Fig. 24b.
vide bearing confinement steel. If rein-
The w^„ force acting against the web
forced bearing confinement steel is nec-
can be transferred by web flexure (see
essary, special reinforcement in accor-
dance with the PCI Design Handbook' Fig. 24c) to the lower web portion of the
is required. beam and then by the lower web portion
to the beam's ends. The height of the
The A, reinforcing of Eq. (12) should
be centered on the load and distributed lower web portion h u. is shown in Figs.
within the ledge over the same distance 24b, 24d, and 24e. The lower web por-
as the A, i, reinforcing bars for inner and tion h. transfers w, to beam ends by
end loads, hut not exceeding b t + h. Bar horizontal flexure as demonstrated in
Fig. 24d. In turn, this horizontal ulti-
spacing should not exceed h nor 18 in,
(457 mm). It is suggested that #3 bars at mate flexural behavior of the lower web
the maximum spacing he provided as a portion can be resisted by reinforce-
minimum for ledge flexural reinforce- ment, which is in addition to that re-
ment. quired for vertical flexure (see Fig. 24e).
Depending upon force magnitudes,
web reinforcement (see Fig. 24c) may
Web Flexure
be necessary to resist the .web flexure
Two instances of web flexure can de- unless:
velop if the spandrel beam's overall tor-
sion equilibrium is generated by the = w2J(2 `) ' 3 A v7' (15)
f^
beam web acting against the top of the w

PCI JOURNAL/March-April 1984 87


SPANDREL Wtu LOCATED 1' BELOW TOP
OF TOPPING OR CURB

Bt TOPPING
NO TOP
T
TEE CONNECTIONS

(a) Construction Arrangement

// Hu

i \ LEDGE
et

h wl >J ;--i------ Hu—WtuL/2

k hh (b) Torsion Equilibrium Forces


Hu

WEB FLEXURE AS
hµ, <_ 2bw+h THIS LEG ONLY
dL Ti
`et/2
Wtu
et

Web Flexure

EFFECTIVE
HORIZONTAL BEAM
TENSION REINF.

(e) Ledge Horizontal Beam

Fig. 24. Torsion equilibrium web flexure.

88
where w, is in lbs per linear foot at ul- the end reaction by following the proce-
timate, e, is in inches, b is in inches, dures of the PCI Design Handbook' for
and A and f are as previously defined. corbels with some modifications.
If fo exceeds 3X til f,', the web rein- Fig. 25 shows the usual variables and
forcementA, per foot of length is: parameters affecting corbel end behav-
ior. In Fig. 25, the end corbel rein-
AS = w (es)
qb d fY
(16)
forcement is given by Fig. 25a for inter-
nal reinforcing bars and by Fig. 25b
which represents a combination of a
where all dimensional units are in steel bearing plate providingA, and the
inches, w,, at ultimate is in lbs per linear internal reinforcement. An elevation of
foot, 0 is 0.85, and fd is in psi. the end corbel reinforcement is illus-
The A, determined by Eq. (16) is ad- trated in Fig, 25c. The corbel A„ steel at
ditive to the ledge attachment rein- the beam bottom is determined from:
forcement, calculated by Eq. (5) and/or
the A,, steel resulting from Eq. (11) due 3 V„l,
A„
= (17)
to common crack planes, but is not ad- 4,4.fYdl
ditive to the web shear reinforcement
which reinforces a different crack plane. or
Usual web torsion steel would not be
required if the spandrel beam's torsional _ V" (18)
equilibrium is provided by the behavior A 91 Of^, 2x2b , d,
depicted in Fig. 24. The spacing of the
whichever is the greater, but a minimum
web A, should nut exceed 18 in. (457
of at least:
mm) nor 3h.
In actual practice, torsional equilib- As, = 0 .08b,d1
rium can be provided by a modified be- f (19)
havior to that shown in Fig. 24. This Y

modified behavior follows when the where in the above equations A„ 1 is in


spandrel beam has adequate torsion square inches, A is as previously de-
reinforcement, proper bottom H,, overall fined, q5 is 0.85, f„ is in ksi, V., is in kips,
torsion equilibrium connections (see and all dimensions are in inches.
Fig. 24b), and the top connection is at The additional ledge reinforcement
the topping level (see Fig. 24a), but only A nh , using identical notation, is:
a limited portion of the topping at the
beam end is involved in developing H,,. A cn
V2
^ (20)
Using the topping to provide one of of,4x2b,d,
the horizontal force components for
overall beam torsion equilibrium re- or a minimum of at least:
quires special attention to the connec-
tion of the column to the topping. This 0 .04 b1d,
(21)
consideration is discussed more fully in ADn = I'd
a following part of the paper.
Reinforcement A„ in the beam web at
Corbel End Behavior the corbel end (see Fig. 25a) is required
to deliver the corbel moment. This
When the end support reaction of a
reinforcing steel can be approximately
spandrel beam aligns with the applied
ledge loads, the ledge acts like an up- selected using:
side down corbel as previously dis-
cussed for Fig. 13. The upside down A.f = dt (A.,) (22)
ledge corbel can be designed to support

PCI JOURNAL/March-April 1984 89


^b w

d2

A S2 THIS _ L
LEG ONLY AT END ONLY

A1—
I
3/4 p 3^a l P
OTHER REINF.
Ip SAME AS (g) P

Avh 1

d^ d1=h

As ) (WELDED
CROSS-BARS)
4 A51
(STEEL PLATE)
I
R R
(a) (b)
11.5

A vh P

çztir y h
__
2h PACIN
@ h/2 MAX. -
^u
Vu= bids ^800p51

(c)

Fig. 25. Corbel end reinforcement.

90
where all variables are in the same rotations, then it may be necessary to
units. consider the influence of principle axes
Proper distribution of the reinforce- of inertia relative to service loads. Span-
ment (Fig. 25) is important. For exam- drel beams usually do not have sym-
ple, A, 1 and A,2 should be placed within metry about either axis. And, if the
1.5h. The required A^,,, should likewise depth of the beam is shallow, then when
be located within 1.5h but also should determining elastic stresses at service
be distributed from the beam end to a level, for either reinforcing bars or pre-
distance of 2h beyond the beam reaction stressed reinforcement, the orientation
support, and should not be spaced of the principle axes possibly can be
greater than h12. Attention must he paid critical. Fig. 26 presents the concept of
to all steel anchorages to insure that the principle axes of inertia found in most
bars will develop their strength at criti- textbooks on strength of materials.
cal sections. The steel plate in Fig. 25b, The influence of principle axes of in-
if used to provide A 1 , needs the correct ertia can typically be neglected for deep
number of'headed studs for anchorage to spandrel beams. However, for small
develop the plate strength. depth spandrels, and particularly those
Experience indicates that the unit end employing prestressing, principle mo-
corbel shear stress at ultimate should ments of inertia may require considera-
not generally exceed 800 psi (5.5 MPa). tion. The orientation of the principle
Also, the spandrel web beyond 1.5h axes is:
from the beam's end should be designed
2I,
to resist the total torsion developed by B – ya tan ' f 1 (23)
the end reaction acting eccentric to the
shear center.
where the angle B, in degrees, is posi-
When corbel end behavior is used to
tive counterclockwise for the dimen-
provide overall torsion equilibrium,
sional orientation (the ledge to the right
careful evaluation must be given to
side) given in Fig. 26. The principle
nonuniform bearing stresses at the ledge
face, the bearing pad, and the support- moments of inertia are:
ing column member. If bearing pads are z
used where corbel end behavior pro- [mar = S y+
r 2 'l ) + J 2
(2
vides overall torsion equilibrium, tor- ,
sional roll of the beam may result from (24)
the pad's deformation depending on the
type of bearing pad employed. and

-F
Beam Flexure Ima„ = Ir 2
2y + 12
[ffl
General spandrel beam flexure in-
volves two distinct loading conditions - (25)
one at service level and the other at ul-
timate state. Depending upon the Flexural analysis of spandrel beams at
beam's cross-sectional dimensions, and ultimate should fellow the requirements
whether or not the spandrel employs of ACT 318-77, Chapter 10. The analytic
reinforcing bars or prestressing strands, procedures can be based upon approxi-
the methods of analysis can be basically mate methods or the specific require-
the same or fundamentally different. ments pertaining to strain compatability
If connections between the spandrel where it is desired to account for all the
beam and the structural units supported reinforcements relative to their dis-
by the spandrel do not prevent torsional tributions in both the tension and com-

PCI JOURNAL'March-April 1984 91


Y

Yp y^+e

,p
C.G.
+8
X -X

Xp

Yl Y p

XpAND YpDEFINE1
_ IMAX.
ORIENTATION OF PRINCIPLE)AXES Sbp Ybp

Fig. 26. Orientation of prrindpll of inertia.

pression zones. When making an ulti- connection design are understanding


mate flexural analysis, consideration the concepts, accounting for all connec-
must be given to those longitudinal tion forces, and evaluating the beam's
reinforcements which are required to loading history from the time it is first
provide for torsion and torsional erected through application of in-use
equilibrium, and therefore cannot con- service (or ultimate) loads.
tribute to the flexural strength. This discussion on spandrel connec-
tions is intended to supplement the PCI
connection design requirements by re-
CONSIDERATIONS FOR viewing the concepts and parameters
CONNECTION DESIGN involved. Discussions on connections
herein relate to connection systems,
Spandrel beam connection design is connection materials, connection inter-
similar to the design for other precast facing, frame moment connections, in-
prestressed concrete connections. All of ternal connection reinforcement, and
the PCI requirements'.' apply to con- connection loads resulting from equilib-
nections associated with spandrel rium requirements of the column sup-
beams. The critical aspects of spandrel porting the spandrel beam.

92
Connection Systems plate's yield stress force by controlling
the cross section of the attaching plate
Spandrel connection systems can be and placing welds only at the plates'
separated into three groups. These ends to allow for deformation strain be-
groups are the basic beam-to-column yond yield. The attachment plate width,
connections, connections between the plus holes in the plate if required to
spandrel and the members supported by limit its tension yield capacity, and
the beam, and those connections re- thickness should only be that necessary
quired at the time of erection. to resist the torsion couple shear force. If
Some of the more common spandrel the building spandrel-to-beam detail is
beam-to-column arrangements are similar to Fig. 27c, torsion equilibrium
shown in Fig. 27. The conditions of connections are not required.
Figs. 27a and 27b result in non-align- In Fig. 28b, the column is shown as
ment of applied ledge loads and the col- projecting to the top of the spandrel
umn reaction thereby requiring con- beam rather than having the beams bear
nections to achieve overall torsion upon the column top. The projection of
equilibrium. The equilibrium connec- the column to the top of the beam is re-
tions use inserts and bearing pads to de- quired to achieve the spandrel torsion
velop the horizontal force couples. The equilibrium in addition to providing the
conditions reflected in Figs. 27a and 27b column top with the necessary horizon-
are for parking garages where the curb is tal reaction for its lateral equilibrium.
also part of the overall connection sys- Structural design requirements for
tem and influences the volume change in-place conditions, or for temporary
forces induced into the connections, erection stability and equilibrium, can
column lateral equilibrium reactions, dictate the need for connections be-
and requirements for erection connec- tween the spandrel beam and the mem-
tions. hers the spandrel supports. An example
Figs. 27c and 27d depict spandrel end of this type of spandrel connection is
connections where the applied ledge shown in Fig. 29. Regarding the attach-
loads and column reactions align. The ment plate connections of Fig. 28, the
dap of Fig. 27d uses a modification to plate cross section (Fig. 29) should be
the beam end to insure load and reaction limited to control the magnitude of the
alignment. This modification requires volume change or other structural de-
careful consideration of the tee leg formation tensile forces which can be
spacing [5 ft (1.52 m) spacings typically] attracted to the connection. Generally, a
and layout of the tees, and its use is gen- i/a in. thick plate, 2 in. wide and 4 in.
erally limited to a maximum column di- long (6.3 x 51 x 102 mm), can be used for
mension of 18 in. (457 mm) for the col- A36 steel which limits the attachment
umn face shown in the Fig. 27d eleva- plate's yield force to 18 kips (80 kN).
tion. This type of controlled yield design is
Spandrel beam-to-column arrange- achieved with welds only at the plate
ments for a building, shown in Fig. 28, ends to insure adequate deformation
require overall torsion equilibrium con- beyond initial yield. An alternate
nections. The top connections used at method of achieving the same result
the floor and roof levels employ welds would be to use an angle welded only at
requiring special details to limit the its toes. The attachment plate of Fig. 29,
magnitude of the volume change forces when in tension due to volume changes
to the connections. When using top (N. forces), induces torsion into the
welded torsion equilibrium connec- spandrel that is additive to the torsion
tions, the magnitude of the volume created by the gravity loads.
change force can be limited to the Erection connections can be part of

PCI JOURNAL/March-April 1984 93


BEARING
CONNECTION
li
TOP OF CURB
II OR TQ ING CURB (SHOWN)

x
II TENSION
CONNECTION

1• BEARING
PAD

Elevation Section

(a)

IjF .
{
0L ^_ TENSION
CONNECTION
x

I I

I ^

I--

BEARING PAD
I^1 BEARING CONN.
V
- ^n OR WELDED
(OMIT IF WELDED)
PLATE CONNECTION

Elevation Section

(b)

Fig. 27. Parking garage beam-column connections.

94
__ _ I
STEEL SECTION
CONCRETE
ENCASED IN
CORBEL
V CONCRETE

Elevation Section

(C)

8" MAX.

Elevation Section

(d)

Fag. 27 (cont.). Parking garage beam-column connections.

PCI JOURNAL/March-April 1984 95


WELD
PLAN OF
LOCATIONS
TOPPING

BEARING P AD ^AIJpLE WELDED


TO BEAM

Elevation Section

(a) Floor Level

WELD
LOCATIONS PLAN OF FE

TEES (NOT
SHOWN)

Elevation Section
(b) Roof Level
Fig, 28. Building spandrels_

96
WELDS AT ENDS

CURB
HAIRPIN REBAR
EACH STEM TOPPING
INTO CURB

TEE FLANGE

Fig. 29. Tee to spandrel connection.

the final permanent connection system, Connection Materials


or they can be connections which are
temporary and serve only during the A variety of readily available materials
erection process. The erection connec- can be used to satisfy the forces and the
tion, if temporary, can remain in-place behavior characteristics of spandrel
and be hidden within later construction beam connections. Among some of the
(such as topping, curbs, and above ceil- common materials used in spandrel or
ings), left exposed to view, or removed. other connection assemblies are bearing
Erection connections that are not re- pads, concrete inserts, steel plates,
moved also can serve as secondary con- welds, coil or threaded rods, reinforcing
nections tying the spandrel to the col- bars or stud type anchors, and concrete.
umn. Bearing pads are used to transmit the
Erection connections are needed to spandrel reaction to the supporting col-
satisfy overall torsion equilibrium, for umn or to transfer loads to the spandrel
providing necessary structural ties he- ledge from the members supported by
tween beams and columns and beams to the beam. Pads likewise are employed
tees (slab type members), and to de- in torsion equilibrium connections (see
velop the stability of the erection brac- Fig. 27). The main functions of hearing
ing. The stability requirements of the pads are to provide uniform bearing by
framing system during erection can in- the pad's ability to accommodate non-
duce forces into connections which are parallel bearing interfacing surfaces and
greater at the time of erection than the to minimize the development of volume
ultimate in-place Ioads to the completed change forces because of the pad's shear
frame. The design of erection connec- deformation characteristics. Most bear-
tions for spandrels and other members ing pads (rubber, neoprene, cotton duck,
generally requires a more comprehen- and fabric reinforced rubber types) can-
sive understanding of structural design not be relied upon to develop torsional
and the criteria for the design than does equilibrium couple forces if the couple
just the design hr final in-place condi- force depends on the shear behavior or
tions. strength of the pad.

PC! JOURNAL, March-April 1984 97

I
Concrete inserts can be used to satisfy tures at the time of welding, type of
many different connection functions. weld, weld heat distortion, weld shrink-
Inserts commonly are employed to pro- age (cooling), and inspection (quality
vide the torsion equilibrium tension control) all must be considered.
farce couple as illustrated in Fig. 27, or Coil and threaded rods are selected to
to tie spandrel beams and columns to a make many connection types. The
horizontal diaphragm using threaded coarse threaded coil rods find wide use
rods into concrete topping, curbs, or in connecting spandrels to concrete
where concrete is the connecting me- curbs and topping, in addition to con-
dium, When selecting inserts for use in necting columns supporting spandrels
spandrels, the designer must consider to horizontal diaphragms. Coil and
whether the insert's capacity is con- threaded rods are used with concrete in-
trolled by the mechanical properties of serts requiring the insert capacity to he
the insert steel or the insert's concrete matched with the capacity of the rod.
shear cone (full or partial). Coil rods, generally ½and 3 in, (12.7 and
Metal plates have a multitude of uses 19 mm) in diameter, are most frequently
in spandrel beams. The plates can serve selected for connections in cast-in-place
as a means to transfer or develop bear- concrete because oftheir coarse threads.
ings, tension, compression, or shear Reinforcing bars are the main part of a
forces. These plates can be made of A36 large number of spandrel connections
steel, higher strength steels, or stainless such as ledges, webs, bearing, daps, and
steels (generally AISI 304). Plates by plate anchorages. Alternately, deformed
definition can mean flat plates, bent bar anchors or headed concrete anchors
plates, or angles. can serve the same spandrel connection
The use of plates requires considera- purposes. When using reinforcing bars
tion of environmental factors relative to for connections, attention must be paid
possible corrosion. Corrosion evalua- to the influence of the bend radius (no
tions can result in the plate(s) not having rebar is bent at a right angle), weld re-
any protective coating, having properly quirements (bar chemical composition,
selected paints (primers and second low hydrogen electrodes, and pre-heat-
coats) and surface preparations, ? being ing), the bar configuration, and place-
galvanized (per ASTM A153) with the ment tolerances.
necessary galvanized thickness, or Concrete is often selected for con-
being stainless steel. Plates used in necting spandrels to other structural
parking garages, plates exposed to members. Typically, cast-in-place con-
moisture, or plates exposed to salt water crete curbs and topping serve not only
air (coastal regions), should be galva- as the connection load transfer
nized at a minimum. Plates used in other mechanism but also constitute the
more aggressive conditions should be of means to prevent axial volume change
stainless steel or be a suitable metal ca- forces from being induced into the con-
pable of resisting the specific chemical nections between spandrels and col-
attack. umns. The use of concrete as a connec-
Welds are important in making span- tion device requires it to have the
drel beam connections. They are used to proper cross section, reinforcement,
fabricate connection hardware, to com- rebar cover, constructability, strength,
plete connections between concrete durability, and mix quality.
embedded plates, and to attach rein-
forcing bars to plates. If welds are to
serve their intended purpose, the type Connection Interfacing
of the weld electrode, pre-heating re- Similar to all precast concrete con-
quirements, environmental tempera- nections, spandrel beams require inter-

98
facing of the connection(s) with other • The tolerance variations in dimen-
parts of the beam and other structural sions a, b, and c can be in opposite
members. Connection interfacing deals directions and, combined with in-
with tolerances, clearances, dimen- sert and beam hole centerline
sional variations, skews, and position skew, thus result in a connection
conflicts between connections and re- that cannot physically be made
inforcements within the beam. even though all tolerances are sat-
Recommended tolerances for con- isfied.
nections are given in the PCI Design The foregoing pertains to any span-
Handbook. € But, when considering tol- drel connection and connections in gen-
erances, evaluations must be made re- eral (such as precast, steel, cast-in-place
garding the cumulative effect of all the concrete, and masonry). Normally, tol-
tolerances involved. The critical tension erance variations of each dimension in-
insert, providing part of the spandrel volved do not occur all at the same time,
overall torsion equilibrium of Figs. 27a and some variations offset others. The
and 27h, serves as an interfacing exam- previous tolerance example regarding
ple. spandrel equilibrium tension inserts
The element which must be adjust- does, however, demonstrate the need
able to accommodate the tolerances in- for an experienced engineer to review
volved is the size of the insert hole in the design when selecting connections
the beam (see Fig. 30). The tolerances and their related clearances.
shown in Fig. 30 that must be reviewed Interfacing tolerances are usually ac-
are: counted for by selecting appropriate
• The as-fabricated location of the clearances. The insert tolerance circle of
column insert as defined by di- Fig. 30 provides an illustration of clear-
mensions a, and b,. The planned ances. The hole diameter D,, can he se-
position ofa, and b, can vary a pos- lected once the insert rod diameter D , is
sible maximum of% to' in. (9.5 to known, insert location tolerances are
12,7 mm), plus or minus. established, and the clearance dimen-
• The a, and b, location dimensions sion D, between the rod and the hole
of the hole in the as-made beam side is determined. A D a of 21/2 in. (63.5
may vary a maximum of % to ' in. mm) results for a tin. (25.4 mm) D 5 , a %
(9.5 to 12.7 mm), plus or minus. in. (9.5 mm) insert tolerance, and clear-
• Skew of the insert in both the x and ance D, of % in. (9.5 mm). Allowing the
y directions that cause the insert beam hole to also have a location toler-
centerline to vary by as much as a ance variation of % in. (9.5 mm) results
ya in. (6.3 mm) at the beam face. in D. becoming 3'/4 in. (82.6 mm).
Similarly, x and y skew of the hole Connection plates embedded into
in the beam can occur. spandrel beams or other precast struc-
• The length of the spandrel beam c, tural members encounter a variety of
can vary up to '/s in. (12.7 mm) or interfacing conditions. Bearing plate
more causing the centerline of the skew, positioning tolerances, and plate
beam hole to change relative to the anchor interference represent the ma-
column insert by the same amount. jority of interfacing concerns.
• The location of the column can be Skewed bearing plates, as shown in
different than planned because of Fig. 31, must be considered when de-
the footing's location or the posi- signing spandrel connections. The skew
tion of anchor bolts within the can result in highly nonuniform bearing
footing. The column erection posi- pad stresses, alter significantly the loca-
tion dimension c, thus can vary by tion of applied loads to ledges or the
yi to 1 in. (12.7 to 25.4 mm). connection supporting the spandrel, and

PCI JOURNALJMarch-April 1984 99


as aC
Y

X X

Cs Y

i e
i ^ I

Cc

(a) Elevation

Y SKEW I I +1/4
SHOWN VARIATION

rb D 45*

Da
/1 Op P
P

INSERT

(c) Tolerance Circle

(b) Section

Fig. 30. Tolerance interfacing.

100
increase N. forces on account of the
plate skew cutting the bearing pad
causing steel to concrete or steel edge
loading with its increased friction coef-
ficient. Concerning the bearing plate
skew, a skew of 1/s to %a in. (3.2 to 4.8
mm) should be considered between any
two corners of the plate.
Plate positioning and the related tol-
erances are influenced by plate anchor
interference. The type, dimensional
size, and the location of anchors should
he evaluated at the time of design to
determine if conflicts exist between any
of the anchors and other materials or
reinforcements within the beam. At a
minimum, plate anchors require a clear-
ance of 3 in. (19 mm) from any other

if
items in the adjacent concrete area un-
less rigid placement tolerances [± Vs in.
^^ 70 ^16
(3.2 mm)] are absolutely assured.
When plates are positioned in beams,
and no anchor interference develops, a
plate position tolerance in plan or ele-
vation of ± 1/z in. (12.7 mm) in any direc-
tion can be expected to occur. Position
(a) Beam End Bearing Skew tolerances for plates apply to plates on
both sides of the connection. It is not
unusual to observe one plate being out
of position by +½ in. (12.7 mm) and the
other by – '/z in. (12.7 mm), creating a
combined misalignment of 1 in. (25.4
mm). Positioning tolerances, like all
other practical real world tolerances, are
cumulative.
An example of cumulative tolerances
compromising the effectiveness of a
spandrel connection is shown in Fig.
32a where it is incorrectly assumed that
LOAD POSITION
both beam and column plates will he
flat, level, and aligned. Details similar to
Fig. 32h should he used since they are
not tolerance sensitive. Proper planning
and consideration of tolerance can elim-
inate most tolerance interfacing prob-
lems.
(b) Ledge Bearing Skew Connection interfacing difficulties
can develop independent of tolerance
Fig. 31. Plate skew. variations. Fig. 33 presents the bearing
pad interfacing between the spandrel
ledge and a long span prestressed dou-

PCI JOURNAtlMarch-Aprik 1984 101


LOOSE WELD PLATE

fL A Ip B

VERTICAL
MISALIGNMENT
OF S%POSSIBLE -

(a) Accumulative Tolerances

(b) Detail Providing Interfacing


Fig. 32. Tolerance interfacing.

102
TEE ENO I.

ROTATION

BEARING PAD
CAMBER - STRESS
ROTATION

BEARING PAD

Fig. 33. Camber interfacing.

hie tee's end camber rotation. The cam- other materials within the beam is only
ber rotation results in nonuniform pad one type of connection component in-
bearing, and depending upon the mag- terfacing. Spandrel beam connections
nitude of the nonuniform stress can employ a range of materials as previ-
damage the bearing pad. Plates cast in ously discussed. All of these materials,
the tee end can be skewed thereby fur- such as inserts and reinforcements, must
ther increasing the nonuniform hearing fit together. Interfacing fit-up problems
stress or exaggerating the camber effect. in most instances are the direct result of
Camber interfacing requires the design inadequate design planning and the
anticipating nonuniform bearing and preparation of improper schematic type
selecting the proper type of bearing pad. details.
A variation of the plate interfacing tol- Proper connection internal fit-up in-
erances is shown in Fig. 34 where terfacing requires the designer to pre-
shared bearing develops. Shared bear- pare details showing all materials which
ing consists of the bearing pad contact- exist at the connection location. Further,
ing both the concrete and steel in the the details must illustrate the items oc-
hearing area. The use of shared bearing curring in the plane of the detail in ad-
can he satisfactory only when interfac- dition to other items perpendicular to
ing is perfect. Commonly, shared bear- the detail plane. Connection fit-up in-
ing creates distortions and distress in terfacing requires three-dimensional
hearing pads, thereby reducing their evaluations of all materials of the con-
effectiveness or causing damage. Con- nection. ACI 315-74, 8 Chapter 6, re-
ditions which can result in shared garding design guidelines provides an
bearing should always he avoided. Fig. excellent summary of interfacing.
34a relates to shared bearing difficulties
while Fig. 34h shows the method to
prevent it. The common bearing of Fig. Frame Connections
34b employs a plate in the beam's end Frame connections between spandrel
larger than the one in the column corbel. beams and columns can be achieved by
Interference of plate anchors with post-tensioning, reinforcement and

PCI JOURNALJMarch-April 1984 103


BEAM END -

STEEL {^

BEARING PAD

COLUMN
STEEL I. -

(a) Shared Bearing

r ^L

(b) Common Bearing


Fig. 34. Shared support.
cast-in-place concrete, or welding. The constituting the frame connections (Fig.
usual method is by welding, and hence 35) are illustrated in Fig. 36. The angle
only welding will be discussed. plate (Fig. 36a) requires not only rein-
A basic criterion of precast frame con- forcing bars to resist the H force, but also
nections is to use only those needed and additional bars to counter the moment
no more. This criterion is based upon created by the eccentricity of the H
the greater costs associated with frame forces, The vertical or horizontal plate
connections and the volume change (Fig. 36b) welded to the embedment
forces attracted to the connections be- should employ a structural steel tee or a
cause of increased column and beam flat plate with stiffeners. The tee stein or
fixity stiffness. Two types of full frame stiffeners are necessary to transfer the
beam-to-column moment connections loose plate force directly to the rein-
am given in Fig. 35. Both types can pro- forcing bars of the embedment.
vide the same moment capacity. Moreover, the distribution of the loose
The flat plate type (see Fig. 35a) has plate force may not he even to the em-
several disadvantages as discussed bedded reinforcing bars considering
below, Welding destroys whatever cor- tolerance variations such as dimensional
rosion protective coating has been offsets in the placement of the connec-
applied, thus requiring field touch-up tion hardware in the beam and column.
which does not have the same protective The use of welded frame connections
ability as the original coating. Also, the emphasizes the need to consider the se-
flat plate connection results in hard quence of the beam-to-column welded
bearing creating the possible need of connections and at what stage of the
some internal bearing reinforcement to construction these connections will be
provide for bearing stress concen- made. Typically, if the beam is sub-
trations, The top connection pocket jected to gravity loads, the frame lateral
shown in Fig. 35a requires a filler, and stability connections should not be
based on actual experience the filler made until all or most of the dead load is
needs periodic maintenance. in place. The sequence of making the
The vertical plate connection (Fig. beam-to-column frame welded connec-
35b) employs a 4 x 6 in. or 4 x 8 in. (102 x tions should be that which minimizes
152 min or 102 x 203 mm) grout column the frame's volume change forces re-
to provide for shear transfer and also sulting from weld shrinkage.
give corrosion protection to the weld Structures which use spandrels or
plates. The grout shear keys minimize other beams in a moment frame to resist
reaction shear to the vertical connection lateral loads and to provide the neces-
plates which can result from frame ac- sary lateral stability require their
tion or elastic and creep deformations of welded frame connections to be prop-
the thin [ 1/4 to % in. (6.3 to 9.5 mm)] erly inspected. The weld inspection and
bearing pad. A drawback to the vertical related testing must be applied to the
plate connection is the dimensional parts embedded in the beams and col-
variation in the 4-in. (102 mm) gap umns in addition to the field welds, If
caused by member and erection toler- rigorous welding inspection and testing
ances. Back-up bars used with full pen- are not employed, welded frame con-
etration welds allow for a dimensional nections should not be used. Likewise,
variation of V in. (6.3 mm). Different proper specifications are required to
sized plates are required to accommo- determine the welding requirements
date the tolerance variations [ viz., 3½, 4, and procedures as controlled by the
and 4½-in. (89, 102, and 114 mm) plate reinforcing bar chemical composition
widths] . and weld electrodes employed.
The details of the embedded plates Frame analysis to determine forces in

PCI JOURNA11March-April 1984 105


EPDXY FILL OR _
EPOxY GROUT - _~`-
BLOCKOUT

It

(a) Flat Plate Connection

SHEAR KEYS

THIN BEARING PAD


SECTION

(b) Vertical Plate Connection


Fig. 35. Welded frame connections.

106
-D
C)
^
V
C
Z
C LOOSE _____ H^2 OR o.aH

C)
H
2 BARS
H
HACK-UP (Q ^^ H,,, OR O.6H
ca
OD

F=H1 TOLERANCE VARIATIONS N STRUCTURAL


POSSIBLE STEEL TEE

(a) Angie Plate (b) Vertical or Horizontal Plate

q I Fig. 36. Frame weld plate details.


the beams and columns, and to calculate proper anchorage must he insured as
the frame deformations, must consider shown in Fig. 38b. Anchorage for bear-
the effect of the various member sizes. ing can be achieved either by welded
This is necessary if the proper stiffness cross-bars or alternate methods using
of the column is to be correctly reinforcing bars bent and placed in a
evaluated. Figs. 37a and 37h reflect the manner that will provide the positive
computer analysis models for the Fig. 35 anchorage. Often, supplemental bearing
frame connections. The more conve- reinforcement is supplied using plates
nient analysis model of Fig. 37c cannot and deformed bar anchors, as given in
be used since it is incorrect and will Fig. 39, to develop the required con-
produce erroneous results. For example, fined bearing capacity.
if the floor-to-floor dimension of the col- The location of the ledge connection
umn is 10 ft (3.05 m), the column length reinforcement and all other spandrel
for Fig. 37c would he 10 ft (3.05 m) connection reinforcing bars is important
whereas the clear column height of Figs. to the connection in developing its de-
37a and 37b could be only 6.5 ft (2 m). A sign strength. Therefore, specified di-
difference in the column height of 3.5 ft mensions for positioning the reinforce-
(1.1 m) has a significant influence on the ment are essential. Along with provid-
magnitude of the volume change forces ing the steel's location, specific po-
induced into the frame connections, and sitioning tolerances must be deter-
upon the beam and column forces. mined. A realistic tolerance for control-
ling critical rebar positions should be
±' in. (6.3 mm) as shown in Fig. 38.
Reinforcement Considerations The concrete cover dimension c for
Spandrel beam ledge and end support precast spandrel beams produced
connections use internal reinforcement under plant controlled conditions and
in connections. The reinforcement exposed to weather should not be less
across critical crack planes and for than 1i in. (31.8 mm) 3 for #5 bars and
bearings must have proper anchorage smaller. However, if the spandrel is ex-
and be correctly oriented if it is to per- posed to a severe corrosive environment
form its function. Additionally, the con- (chloride salts) the minimum cover
nection steel requires more accurate should he 2% in. (57.1 mm) forconnection
placement than the usual beam flexural rebars if a ' in. (6.3 mm) tolerance is
and shear reinforcement. And, if the specified. Also, severe exposure may
spandrel is used in an exterior environ- require the spandrel's concrete to have a
ment, concrete cover and reinforcement maximum water-cement ratio of 0.4 by
corrosion demand increased attention. weight 9 to provide the necessary corro-
Ledge reinforcement, as shown in Fig. sion protection for steel.
38, can serve solely as flexural steel or as The design planning of re info rcernent
combined flexural and confined bearing for connections and other reinforcing
steel, The reinforcement of Fig. 38a bars in spandrel beams requires equal or
should be considered only in providing greater skill than the reinforcement
for ledge flexure, and the bearing design based on structural design. The span-
for V. and N. should employ plain con- drel's reinforcement design, selection of
crete analysis methods since the critical bar sizes, bend radius, concrete cover
bearing failure crack plane can by-pass requirements, and interfacing of all the
the reinforcing steel because of its end beam's steel can control or dictate the
bend radius. If the ledge reinforcement spandrel beam's cross-sectional dimen-
is also to accommodate bearing (ulti- sions as can the width of the compres-
mate bearing stresses exceed that al- sion flange relative to the beam's span
lowed by plain concrete bearing), (width = span/50).

108
COLUMN

3 //JOINT NO. 3
\® / MEMBER NO.

^} ^5 1r ^ 1 Ib
BEAM 20Ib

2 2

(a) Fig. 35a Model

COLUMN

QQ 7 Ib
7 6
201b
b [^j 1 1 Yb

5®2 C BEAM 5 Ib O Ib

®4 3® Ib 30lb
Q 4

(b) Fig. 35b Model

a= DISTANCE 0, COLUMN 3
TO BEAM REACTION

b- DISTANCE BETWEEN 1
CONNECTIONS

C-DISTANCE t BEAM
TO BEAM BOTTOM Q

(c) Centerline Model

Fig. 37. Computer analysis models.

PCI JOURNAL/March-April 1964 109


vu

N
C}I/4•
BEARING CRACK
P
POSITIONING
y (PLAIN BEARING)
BARRS
CRITICAL CRACK
PLANES

C C f '/4 °

(a) Ledge Flexure Only

VU WELDE
CROSS-BAR

3 ►u -- c±t14'

POSITIONING BEARING CRACK Alternate Ledge


BARS (REINF. BEARING)
Rebar Details
CRITICAL CRACK
PLANES

(b) Ledge Flexure & Bearing

Fig. 38. Ledge reinforcement.

110
vu

Ne

C C

Fig. 39. Supplemental ledge bearing reinforcement.

Fig. 40 shows a typical arrangement of tolerance. Tolerances for locating the


reinforcement. It becomes immediately various bars within the cage and within
apparent that concrete cover and bend- the beam must be considered, as must
ing radius limit the locations of other the bars' having an outside diameter of
steels. The 2-in. (51 mm) grid for locat- Vs in. (3.2 mm) greater than their normal
ing the prestressing strands is not com- size on account of the ribs. Fabrication
pletely available. Study of reinforcing tolerances for individual bent bars must
bar bends can be important because the also he evaluated. The fabrication toler-
bar bends influence where other rein- ance for stirrups and ledge A. bars is Vs
forcing bars can be positioned. Another in. (12.7 mm) on specified dimensions
reinforcing bar consideration relates to plus another Vs in. (12.7 mm) for out of
some bars being necessary to secure or square configuration.
provide for dimensional positioning of The preceding discussion on spandrel
key steel, such as the ledge A s bars. reinforcements, and the many criteria
An additional factor to be evaluated the bars must satisfy, emphasizes the
regarding the influence of the rein- fact that the designer must select the
forcement dictating, in part, the po- three-dimensional reinforcements and
sitioning of the prestressing strands (see make sure his design can actually be
Fig. 40) is the non-alignment of the built. A cause of many spandrel beam
strand's centroid with respect to the problems and distresses is that beam
spandrel's principle Y, axis (see Fig. reinforcements are selected without any
26). This non-alignment can result in concern for interfacing between them,
horizontal sweep of the beam. and critical reinforcements end up sig-
Fig. 40 shows reinforcement ar- nificantly out of position.
rangements without any provisions for The reinforcement considerations

PCI JOURNAUMarch-April 1984 111


10"

A t TORSION BARS --" - AV, A t OR A sh BARS

1
11J2COVER TYP g
SHOWN _ LEDGE A5 BAR

POSITION BARS & / 3/4k BEARING


A t BARS CHAMFER

CS

d f •• `" Ah '12'

CAGE AND /OR P/S STRANDS


BEAM A 5 BARS
2 STRAND GRID

Fig. 40. Reinforcement interfacing.

presented herein relate to ledge and The influence of column equilibrium


web reinforcement alone. The rein- loads on the spandrel torsion stability
forcement concepts discussed apply connections is even more pronounced if
equally to spandrel beam ends with re- no direct connection is made between
gard to daps, ledge corbel behavior, or the column and the horizontal dia-
beam end torsion. Typical connection phragm (floor or roof level). In this in-
reinforcement details can only he typi- stance, additional horizontal forces are
cal when they interface with all rein- induced into the beam end torsion
forcement conditions, equilibrium connections which can rep-
resent a marked increase in the required
Column Influences connection design load. Spandrel con-
Connections which attach the span- nection designs which do not consider
drel beam to the column for the basic the additional column equilibrium
purpose of supplying the beam's overall forces have resulted in connection fail-
torsion equilibrium can be subjected to ures and expensive connection repairs.
additional column equilibrium loads. Depending upon the framing ar-

112
rangement of the spandrels to the col- GOOD DESIGN PRACTICE
umn, the analyses to determine beam
end connection forces can be very com- The design of spandrel beams en-
plex. Fig. 41a illustrates the framing and compasses not only an understanding of
pertinent dimensions for a two-story engineering fundamentals but also
parking garage where the spandrel properly requires an appreciation of
beam torsional equilibrium is provided practicality and experience. The fol-
by end connections. In addition to the lowing outline on general design
end connections, the column is attached guidelines for spandrels is based in
directly to the horizontal diaphragms as large part upon observations of past
is the beam for its lateral support. The performance, both good and bad. Among
diaphragm horizontal connections are the many aspects involved in spandrel
achieved with inserts, coil rods, and beam design, those set forth represent
concrete topping. The horizontal dia- some of the more pertinent factors.
phragm is in turn laterally supported by
shear walls or moment frames. Cross- Sectional Dimensions
The analysis to determine the con- • The spandrel web width generally
nection forces resulting from both beam should not be less than 10 in. (254
torsion equilibrium and column equilib- mm), and never less than 8 in. (203
rium is best made using computer meth- mm).
ods. The analytical model and require- • The horizontal ledge projection
ments of the model to duplicate the ac- should not be less than 6 in. (152
tual frame behavior are presented in mm).
Fig. 41b. The spandrel beam vertical • The vertical height of the ledge
span properties (see Fig. 41c) can be should not be less than 10 in. (254
determined from the web width of 8 in. mm), and for beams supporting
(203 mm) acting over the lesser hori- large concentrated reactions a
zontal length of eight times the web 14-in, (356 mm) height or greater is
width or the total height of the spandrel
realistic.
beam. • The final beam cross-sectional di-
The model (Fig. 41b) deals only with mensions may be dictated by the
the additional column equilibrium needs of practical non-congested
forces to which the beam-to-column reinforcement arrangements to in-
connections are subjected. Another sure the spandrel can be realis-
loading condition which must he re- tically built within its selected di-
viewed is that resulting from volume mensions.
change deformations acting parallel to • Blockout and connection fit-up re-
the tee span. The beam-to-column con- quirements in spandrels, particu-
nections can be considered as not re- larly moment frame beams, may
ceiving any additional loadings from control the cross-sectional dimen-
volume change movements providing a sions if all internal components are
direct column-to-diaphragm connection
to he properly integrated within
exists. However, when no column-to- the beam's final dimensions.
diaphragm connection is used, the ad-
ditional connection forces must be ac-
counted for. Instead of the rigid hori- Reinforcement
zontal supports of Fig. 41b, horizontal • Bars have a radius at bends, and are
framing members, per Fig. 42, should be not bent at a sharp 90-deg angle.
used to simulate the volume change • Fabricated reinforcing bar config-
deformations so the total forces to the orations have tolerances and usu-
connections can be determined. ally will be out of square.

PCI JOURNAL/March-April 1984 113


11181
CURB

TOPPING
F

^- 4^4 B* 5

° 24 DOUBLE TEE °p COLUMN


C7 HORIZ. TIE

BEAM R't O
HORIZ. TIE

`p
O 18'x 18 COL. ° 8w
AT 24'- 0' o
COLUMN

t, CURB TIED TO BEAM


AND COLUMN BY
GRADE SLAB THREADED RODS

- --- Rt - TEE STEM REACTION


iy Rb = SEAM END REACTION

(a) Framing Arrangement

Fig. 41. Connection loads induced by column equilibrium.

114
COLUMN

18 10
BEAM Ft CONNECTION

U I
FLOOR DIAPHRAGM 17 9 FLOOR DIAPHRAGM

BEAM HORIZONTAL SUPPORT oT H COLUMN HORIZONTAL SUPPORT

16 Q a
Y S P G0.25
15 T

1d
L 6 BEAM F 1 CONNECTION
p- 0 E —( SPANDREL END REACTION
N1
FLOOR DIAPHRAGM 1 13 5 1FLOOR DIAPHRAGM
BEAM HORIZONTAL SUPPORT ECOLUMN HORIZONTAL SUPPORT
N D N
N 12 K 4 C+i - BEAM Ft CONNECTION
M
BEAM VERTICAL
11 a ^n
n! SUPPORT CONNECTION
0.50

1/2 COLUMN PER


SPANDREL BEAM END
B

-TOP/FOOTING LEVEL

2
IMAGINARY COLUMN LENGTH
TO SIMULATE PARTIAL
A FIXITY RESULTING FROM
FOOTING (20 TO 50% FIXED
1 TO 18 - JOINT NO. 1 COL. BASE)
A TO R - MEMBER NO.

(b) Computer Analysis Model

Fig. 41 (cont.). Model for analyzing connection loads induced by column equilibrium.

PCI JOURNAL/March-April 1984 115


SUPPORT JOINTS JOINT LOADS
(Service Loads)
Jt. No. Type Jt. No. Loads
I pin - y roller II BP=56Kand
2 pin - no rollers Couple* = 28 Ft.-K
5 pin - y roller IS EP=56Kand
9 pin - y roller Couple* = 28 Ft.-K
13 pin - y roller * Couple acts counter-clockwise
17 pin - y roller
MEMBER PROPERTIES
Member End Jts. I - in4 A - in 2 Joint Releases
A 1,2 8748/2 324/2 none
8 2,3 8748/2 32412 none
C 3,4 8148/2 324/2 none
D 4,5 874812 324/2 none
E 5,6 8748/2 324/2 none
F 6,7 8748/2 324/2 none
G 7,8 8748/2 324/2 none
H 8,9 8748/2 324/2 none
1 9,10 874812 324/2 none
J 3,11 9()00* 0.1** none
K 4,12 0.1** 9000* pin @ each end
L 6,14 0.1** 9000* pin @ each end
M 11,12 2731 512 pin @ I
N 12,13 2731 512 none
0 13,14 2731 512 none
P 7,15 9000* 0.1** none
a 8,16 0.1** 9000* pin @ each end
R 10,18 0.I** 9000* pin each end
S 15,16 2731 512 pin (al 16
T 16,17 2731 512 none
U 17,18 2731 512 none

* Arbitrarily selected small to simulate an x-roller or to simulate no flexural


stiffness
** Arbitrarily selected large to simulate axially rigid connection or to simulate
flexural rigidity

(c) Analysis Data

Fig. 41 (cont.). Data used in computer model for analyzing connection


loads induced by column equilibrium.

116
T
16
S
15

14

20

2.25'

lZ

NOTE:

1. MEMBERS V,W,X & Y HAVE ARBITRARILY


LARGE AREAS.

2. VOLUME CHANGE INFLUENCE RESULTS FROM


ASSIGNING A DECREASE IN LENGTH TO
MEMBERS V,W,X & Y WHICH CAUSE A
DISPLACEMENT AT JOINTS 5,9,13 & 17
EQUAL TO THE DESIGN VALUE.

3. REMAINDER THE SAME AS FIG. 41b

Fig. 42. Computer mode! for analyzing connection loads induced by


column equilibrium and volume change deformations.

PCI JOURNAL/March-April 1984 117


• Actual bar diameters, considering • Tolerances generally accumulate
ribs, are about ' in. (3.2 mm) and this effect should be consid-
greater than their nominal size. ered.
• Spandrel internal reinforcement • Designs should be based upon the
for connections typically should be tolerances and clearances selected
#3, #4, ur#5 bars. which produce the greatest forces.
• Details should be prepared during
design to check that the three-
dimensional interfacing require-
Corrosion Protection
ments for reinforcing bars are • Concrete cover to reinforcing bars,
satisified. considering placement tolerances,
• The arrangements and positions of should be selected considering all
the steel must he such that ordinary influencing factors.
workmen can fabricate and con- • Coatings to metal plates should he
struct them. specified in detail and not he refer-
• Clearances of 3/4 in. (19 mm) be- enced as simply rust inhibitive
tween spandrel reinforcements in paint or galvanized.
different planes should be planned • Welding to hardware in beams that
to accommodate reinforcing bar have corrosion protective coatings
tolerances and interfacing. should be done prior to the coating
• Special welding requirements and application.
procedures should be determined • Parking garages using metal plates
whenever welded reinforcing bars exposed to the elements should re-
are used at critical connections. quire all plates to be galvanized at
• Reinforcing bars should not be a minimum.
welded near cold bends, and
should never be tack-welded. Loads
• Loads to spandrel beams and their
Tolerances and Clearances connections during erection may
• A 1-in. (25.4 mm) minimum clear- be greater than the permanent de-
ance between spandrel beams and sign loads.
all other framing members should • The design should consider the
be selected. Long span spandrels, combined effect of the different
spandrels of large depth [48 in, possible loading cases.
(1.22 m) and greater], and span- • The influence of volume change
drels supporting members span- loads axially and rotationally, and
ning 60 ft (18.3 m) or more may other deformation caused loads,
require clearances between the should be accounted for in the de-
sign. All connections should be de-
other structural components of up
to 1 1/2 in, (38 mm) to provide for signed for a minimum N. tension
force equal to 0.15%x.
tolerance fit-up interfacing.
• Separate evaluations of fabrication • Loads to spandrels generated by
the beam's torsional equilibrium
and erection tolerances should be
requirements and by the support-
made for each design rather than
ing column's lateral equilibrium
relying solely on "usual" industry
forces should be determined.
tolerances.
• The influence of structural defor-
mations during erection should be Connections
considered when selecting toler- • Connections or other means must
ance and clearance requirements. be used to provide for spandrel

118
beam overall torsion equilibrium at mum internal reinforcement to re-
the time of erection and for the in- sist N. forces.
place permanent conditions. • Consider using preformed pads
• Where possible, avoid using ten- composed of synthetic fibers and a
sion insert connections to provide rubber body, % to 'A in. (9.5 to 12.7
for spandrel beam torsion equilib- mm) thick, for most bearing condi-
rium because of tolerances. tions.
• Consider all load combinations • Avoid using neoprene pads for
from construction to final in-place general bearing conditions except
forces when designing and detail- when control of volume change
ing spandrel connections. deformations (not at expansion
• Evaluate the influence of toler- joints) is a critical design parame-
ances on all the various spandrel ter, and then use only non-com-
connections. mercial structural neoprene grades
• Do not count on hearing pad shear satisfying AASHTO specifications.
behavior as a means of providing
overall beam torsion equilibrium.
• Select connection details which are Ultimate Load Factors
not tolerance sensitive. • Loads resulting from volume
• Insure that connection hardware change deformations should
can he practically integrated into employ the same ACI Code 1.7
the spandrel without interference load factor as required for live
or conflict with other materials loads.
within the beam. • All spandrel beam connections
• Connections using loose field should use a load factor of at least 2
welded plates or threaded bars (2 = 4/3 the usual combined ulti-
should require the anchoring mate dead and live load factor of
hardware embedded in the beam 1.5).
and/or the adjoining member to • Connections which rely upon con-
have a load capacity one-third crete shear cones for capacity
greater than the loose connecting should use a load factor of 3 unless
plate. a lower load factor of 2.5 can be
• Complete free-body diagrams prudently justified.
should be developed for each con-
nection design to insure that all
forces are accounted for and that Supporting Columns
statics are satisfied. • Loads to the column resulting from
spandrel beam torsion equilibrium
connections and the beam reaction
Bearing Considerations should be analyzed to insure that
• Employ plain concrete bearing in the column is correctly designed.
combination with bearing pads • Column design requires that the
whenever possible. influence of the column's and the
• At service load levels, nominal PIA spandrel's actual size be consid-
bearing stresses for ledges and ered by the analysis rather than just
beam ends should be in the 800 to using simple beam-to-column cen-
1000-psi (5.5 to 6.9 MPa) range. terline dimensions for the analysis.
• Avoid using plates for bearing sur- • Spandrel beams at the uppermost
faces whenever practically possible. level, or elsewhere, should not
• When using plain concrete bear- hear directly upon column tops but
ing, always provide some mini- instead be supported upon column

PCI JOURNAL/March-April 1984 119


haunches or corbels, thereby pro- considered as only a simple com-
viding for the top of the column to ponent design. Rather, there must
be approximately level with the top be "one" in-charge designer to in-
of the beam. sure that the spandrel beam's de-
sign satisfies the overall criteria,
Inspections that building frame interfacing
• Connection reinforcements should structural requirements are met,
be inspected for placement posi- that deformation and restraint con-
tion and arrangement by the de- ditions have been evaluated, se-
signer or his representative during lected tolerances and clearances
spandrel beam fabrication. are satisfactory, and the connec-
• Welded moment frame connection tions have been correctly inte-
hardware embedded in spandrels grated.
(and columns) requires weld in- • The hindsight key to good spandrel
spection and testing of the rebar beam design is that the in-charge
and plate welds. designer he meticulous regarding
• Field welds employed to complete requirements and details.
moment frame connections require The concepts discussed and the re-
weld inspection and testing. quirements reviewed for spandrel beam
design apply equally to all aspects of
precast prestressed concrete design, and
CLOSING COMMENTS to engineering design in general.
The behavior, design considerations,
design relationships, and general
guidelines on spandrel beams as pre- ACKNOWLEDGMENTS
sented are based upon the author's de-
sign, construction, failure investigation, The graphic figures were prepared by
and repair experience. Steven E. Adams of Raths, Raths &
Spandrel beam design, if it is to be Johnson, Inc.
successful, requires consideration of the The author wishes to express his ap-
following items: preciation to the many individuals who
• No magic details or solutions exist. reviewed his manuscript and who
Each project's design requires the supplied many constructive comments.
engineer to evaluate all the cir- In particular, he wishes to thank Alex
cumstances, determine the design Aswad, Kamal R. Chaudhari, Ned M.
criteria, and develop the appropri- Cleland, Arthur R. Meenen, A. Fattah
ate solutions. Shaikh, Edward R. Sturm and H. Carl
• Spandrel beam design cannot be Walker.

Note: Discussion of this paper in invited. Please submit


your discussion to PCI Headquarters by November 1, 1984.

120
REFERENCES
1. PCI Design Handbook—Precast Pre- tober 1980, pp. 32-100.
stressed Concrete, Prestressed Concrete 6. AC! Committee 332, "Building Code Re-
Institute, Chicago, Illinois, 1978, 369 pp. quirements for Structural Plain Concrete
2. PCI Manual on Design of Connections for (ACI 332-72)," American Concrete Insti-
Precast Prestressed Concrete, Prestressed tute, Detroit, Michigan, 1971, 7 pp.
Concrete Institute, Chicago, Illinois, 7. Keane, J. D., Systems and Specifications,
1973, 99 pp. Steel Structures Painting Manual, Steel
3. AC! Committee 318, "Building Code Re- Structures Painting Council, Pittsburgh,
quirements for Reinforced Concrete (ACI Pennsylvania, V. 2 (Third Edition), 1982,
318-77)," American Concrete Institute, 382 pp.
Detroit, Michigan, 1977, 103 pp. 8. AC! Committee 315, "Manual of Standard
4. Roark, R. J., and Young, W. C., Formulas Practice for Detailing Reinforced Con-
for Stress and Strain, Chapter 7, "Beam; crete Structures (ACI 315-74)," Chapter 6,
Flexure of Straight Bars," McGraw-Hill, "Structural Design Details for Buildings,"
Inc., New York, N.Y., 1975 (Fifth Edition), American Concrete Institute, Detroit,
624 pp. Michigan, 1974, pp. 54-59.
5. Collins, M. P., and Mitchell, D., "Shear 9. AC! Committee 201, "Guide to Durable
and Torsion Design of Prestressed and Concrete (ACT 201.2R-77)," American
Non-Prestressed Concrete Beams," PCI Concrete Institute, Detroit, Michigan,
JOURNAL, V. 25, No. 5, September-Oc- 1977, pp. 20-23.

Note: An Appendix giving design examples


is included following the Notation Section.

PCI JOURNAUMarch-April 1984 121


NOTATION
= distance from end of beam to d, = distance from end of beam to
horizontally applied load or end concentrated load closest to
height of ultimate strength rec- beam end, in.
tangular stress block d,' = distance from centroid of tension
AA = longitudinal reinforcement in
reinforcement to extreme ten-
beam ledge near ledge top, scl sion fiber, in.
in.
A, = longitudinal torsion reinforce- d 1
, = distance from non-ledge side of
beam web to centroid of 45 deg
ment about beam perimeter, sq
A,, web reinforcement posi-
in.
A,, = reinforcement to resist flexure tioned closest to ledge side of
beam, in.
and/or direct tension, sq in.
A,,, = hanger shear reinforcement d, = distance from centroid of A + , to
top of ledge for corbel end sup-
across inclined dap crack, sq in.
port, in.
Ad1 = corbel type reinforcement, beam
ledge bottom, at beam end sup- d 2 = distance from beam web face,
port, sq in. ledge side, to centroid of corbel
A, = beam web reinforcement, non- end A,2 vertical web reinforce-
ledge side, at beam end support, ment. in.
srq in. e d = distance from beam end at top of
A, = beam torsion reinforcement, ledge along an inclined 45 deg
closed stirrups, ledge side of line on the web to a 90 deg inter-
web, sq in. secting line passing through the
A, = web shear stirrup reinforcement, H, equilibrium connection cen-
sq in. troid (Fig. 21b), in.
AvA = corbel type reinforcement, beam er = distance from 1 in. (25.4 mm)
ledge, at beam end support, sq below the top of the concrete
in. topping to the top surface of the
A. = beam end 45 deg torsional equi- beam ledge, in.
librium reinforcement ledge e m = perpendicular distance from the
side of web, sq in. H. equilibrium connection cen-
A = beam end longitudinal compo- troid to the inclined 45 deg line
nent of A,, torsional equilibrium along which e Q is measured (Fig.
reinforcement ledge side of 21b), in.
web, sq in. e l = distance from beam ledge load,
A1,,, = beam end vertical component of uniform or concentrated, to the
A,, torsional equilibrium rein- beam shear center, in.
forcement ledge side of web, sq es = distance from beam end reaction
in.
to the beam shear center, in.
bt = bearing width of concentrated
= compressive strength of beam
ledge load, in.
concrete, psi
b,,, = width of.beam web, in.
bl = effective width of ledge corbel fl = beam concrete combined
flexural and tension stress, psi
at beam end support, in.
C = clear concrete cover to reinforce- f = beam web flexure tension stress,
ment, in. psi
d – distance from extreme compres- = yield strength of reinforcement
sion fiber to centroid of flexural or steel, psi or ksi
tension reinforcement, in. h = vertical height of beam ledge, in.

122
h h = vertical distance from 1 in. (25.4N. = ultimate volume change hori-
mm) below concrete topping to zontal load applied to beam, lbs
force centroid of beam H. bot- or kips
tom equilibrium connection at P = concentrated load applied to
beam end (Fig. 24h), in. beam ledge, lbs or kips
h,o = vertical height of beam lower P„ = ultimate concentrated load ap-
portion acting as a horizontal plied to beam ledge, lbs or kips
beam (Figs. 24b and 24d), in. R, R,,, Ra = beam end support reaction,
H = horizontal applied load or hori- lbs or kips
zontal torsion equilibrium con- s = spacing between concentrated
nection force, working or ulti- loads, in.
mate, lbs or kips S bp = principle section modulus at
H. = ultimate beam end horizontal beam bottom, in.3
equilibrium connection force, T, TA, TB = beam end equilibrium
lbs or kips torque, in.-lbs or ft-kips
= maximum principle moment of V„ = ultimate shear load applied to
inertia, in.4 beam ledge, lbs or kips
= minimum principle moment of = ultimate lateral loads per unit
inertia, in.4 length applied to beam for beam
I r = moment of inertia about x axis, torsional equilibrium (Fig. 24),
in.4 lbs per ft
I product of inertia, in.4 Ybp = distance from beam cross-sec-
I„ = moment of inertia about y axis, tional centroid to extreme bot-
in.4
tom fiber along inclined vertical
axis, in.
1„ = ratio of ultimate internal mo-
= coefficient based upon unit
ment resisting arm to d, in.
weight of concrete, 1.0 for nor-
1 p = horizontal projection of beam mal weight and 0.85 for sand-
ledge from web, in. light-weight
L = span ofheam, in. = ultimate strength reduction fac-
m = variable defining length of beam tor, 0.65 for plain concrete ten-
ledge attachment to web de- sion, 0.85 for shear, and 0.90 for
pending upon type and location reinforced flexure
of ledge load = summation symbol
N = volume change horizontal load = angle of inclination of principle
applied to beam, lbs or kips inertia axes

PCI JOURNAilMarch-April 1984 123


APPENDIX-DESIGN EXAMPLES

The following three numerical exam- for an end support corbel plus a sample
ples address the design of a typical calculation for determining the princi-
spandrel beam for a parking garage in- pie moments of inertia for a building
cluding the reinforcement requirements spandrel beam.

EXAMPLE 1 — PARKING GARAGE SPANDREL BEAM

Spandrel Beam Data Beam reinforcement


f„ =60ksi
Normal weight concrete The spandrel beam details are shown in
f,' = 5000 psi the sketch.

CP I'_ Q••
^o"' Io"x 32 "77`
30", (cwT)
/O ~3 "TOPPING

is 14`

S = 4'"
WT o. 84 k I^'

p P P f P P

CLEAR /8's-
-

Spandrel beam details

124

Design Loads
Live Load = Weight of Cars + '/2 Snow
Weight
=50+'/2(30)= 65psf SHEAR
_ GENTEK
Note that there is no live load reduction. dX

Concentrated load per tee stem


(D + L) = 25.3 kips (working) Id

Find Shear Center


Refer to Fig. 20.

W1 = 10 in., W. = 14 in.
Determination of
shear center
h l = 65 in., li e = 11 in.
_
_ 11 11 (14) 3
es 210(65)+ 11 (14)g Beam End Design
= 0.060 in. Refer to Fig. 27b for beam-to-column
details.
ex = + 0.060 = 5.06 in.
Design data:
65 [ 65 (10) 3 1 1. Refer to Fig. 21.
e" 2 L65(i0)a+14(11),) 2. Consider shear center aligns with
web centerline.
= 25.26 in.
3. ZP = 3 (25.3) = 75.9 kips (working)

e„ = 2 + 25.26 = 32.26 in Find ultimate load factor:

Location of beam center of gravity: PD +L = 25.3 kips


PD = 15.4 kips
z = 5.84 in., y = 32.97 in. Py = 9.9 kips

Hu

ffu

N Tp WES 'z

Spandrel beam end design


PCI JOURNAIJMarch-April 1984 125
LF = 1.4D+ 1.7L Am„ + A, = 0.92 + 0.17/2
D+ L = 1.00sgin. (1.65to6.65 ft)
_ L4(15.4)+ 1.7 (9.9) #4 stirrup spacing:
25.3
= 1.52 0.20
= 10.2 in.
1.08/55
Find J1
H a forces to beam web, use LF = 1.52. A,,, bar size per 12 in. spacing:
H
'
1.52 (75.8) 9.5
62
12( 55 )= 0.20sgin.
= 17.7 kips
Use #4 bars at 12 in. center to center.
to top tension insert at ultimate.
Find e d and e: Determine tension insert ultimate
loads:
ed = 2 (50+5)=38.891n. I. Steel ultimate = (4/3) (LF) (Working
Load)
2 ed = 2(38.89)= 77.8 in. 2. Concrete punching shear beam web
and insert shear cone requires a load
0
-f5+5- factor equal to three.
e^ l 2 5 = 31.8 in. 3. H Steel:
Determine A ^, reinforcement: (4/3) (26.8) = 35.7 kips
UseEq.(1).d,,-10- 1.5= 8.5 in,
4. H, Concrete:
A - 17.7(31.8)
U'
0.85 (8.5) 60 2fi.8
. _ 52.9 kips
= 1.30 sq in. (minimum) 3( 1.52
Find minimum A,, and A,1:
Use Eqs. (2) and (3). Summary

A eon = A k,, A=2 =2r


(1.30)
1. Use #4 closed stirrups at 10 in. cen-
ter-to-center over a length of 5 ft from
beam end.
= 0.92 sq in. 2. Use #4 bars for A! at 12 in. center-to-
Distribute 0.92 sq in. horizontally over center about beam perimeter. Use
hairpin bars according to Fig. 25 at
beam end.
2 (2 ea) = 55 in. 3. Note that other design cases may
control reinforcement.
Select#4 stirrups for A.o and A.
Ao from: End to 1.65 R = 0.32 sq injf Ledge Attachment Design
1.65 to 6.65 ft = 0.17sgmill
Determine ultimate stem load:
Note that the above steel area includes
the c, reduction due to torsion. V. = (4/3) (1.52) (25.3) = 51.3 kips
Am „ + Ad = 0.92 + 0.3212 Check end shear V„ for f1 at ultimate:
= 1.08 sq in. (end to 1.65 ft) Use Fig. 22 and Eq. (4). % i = 4.5 in.
126
d emo, = 2h =2(14) =28 in. Use #4 stirrups at 7 in. center-to-center
starting at 4 ft 6 in. from each end.
de = 19.75 in. controls de selection

s,^Rr = bt + 4 h = 4.75 + 4 (28) Summary


= 116.75 in. 1. Consideration should be given to in-
creasing h,o to 12 in. if greater rein-
s = 60 in. controls
forcement spacing is required.
m = d, + s/2 = 19.75 + 6012 2. Note that other design cases may
= 49.75 in. control.

51,300 6(51,300)(10/2 + 4.5) Ledge Load Transfer Design


f` 10 (49.75) + (10)2 49.75
V,, = 51.3 kips per tee stem (see Fig. 23)
691 psi

f i greater than
Concrete shear capacity inner V„

s - 60 in. greater than


3X j7 = 3 (1) 5000 = 212 psi
b t + 2 h = 4.75 i- 2 (14) = 32.75 in.
Therefore, ledge reinforcement is re-
quired. Use Eq. (6)

Select ledge attachment reinforcement X


at end V. Vq-3.8)14(1)U
100
Use Eq. (5):
[2(6) +4.75+ 14] = 77.6 kips
_ 51.3(10+4.5-0.5)
Plain concrete shear capacity is
A^ ^ 0.85(60)(10- 1.5- 0.5) greater than applied V. = 51.3 kips.
= 1.76 sq in. over length m = 49.75 in. Therefore, no reinforcement per Eq.
(11) is required for inner loads.
#4 bar spacing:
Concrete shear capacity end V.:
49.75
d, < 2 li, de = 19.75 in., and 21: = 28 in.
1.7610.20 = 5.65 in.
Use Eq. (8):
Use #4 stirrups at 6 in. center-to-center
over a length 4 ft 6 in. from beam end. 0.85(14)
V = 1x
Determine inner o„ ledge attachment " 1000
reinforcement: 12(6)+ (4.75 + 14)12+ 19.751
= 34.6 kips
V. = 51.3 kips
Plain concrete shear capacity is less
By inspection, if f, > 3 X J ,, reinforce-
than applied V. = 51.3 kips. Therefore,
ment is required.
reinforcement per Eq. (11) is required
Use Eq. (5) and previous example.
for end V,,.
A, = 1.76 sq in. over length m = 60 in. Select end V,, reinforcement:
Check applied shear stress.
#4 bars with a spacing of: Shear Area = h (b, + h)
60 = 14 (4.75 + 14)
= 6.82 in.
1.76/0.20 = 262.5 sq in.

?CI JOURNAUMarch-April 1984 127


c. 51,300 Select two #5 bars with end hairpins.
= 262.5 = 195 psi
Select ledge flexural reinforcement.
This stress is satisfactory since it is less Use Eq. (12):
than lox VT = 707 psi.
Use Eq. (11): d -h - 1.5 = 14- 1.5 = 12.5 in.

_ 51.3 A1=
Agh 0.85 (60) = 1.01 sq in. 0.851 (60)[ 51.3 ( 4 12.5
1
over length bt + h = 18.75 in. + 0.15 (51.3) 14
12.5 J
since d e > (b + h)12.
= 0.53 sq in, per tee stem load.
AYh steel area provided by ledge rein-
lorcement at 6 in. center-to-center for Use #4 bars at 12 in. center-to-center
end V. (see previous calculations) is: with two additional bars at each tee stem
distributed over b, + h = 18.75 in.
0.20 (18.75/6) = 0.62 sq in.
since de > (b + h)/2
Provide 1,01 - 0.62 = 0.39 sq in. of
additional steel reinforcement.
Use two #4 closed stirrups in addition Design Summary
to other reinforcement at each end V. See Fig. 27b for design details. Follow
Determine A h = A,h12 = 1.0112 = 0.50 ACI 318-77 (Chapter 11) for shear and
sq in. torsion reinforcement.

Reinforcement details:

Location from
beam end A, + 2A, A,
0 ft to 1 ft 7Y, in. #4 bars at 5.6 in. #4 bars at 12 in.*
1 ft 73/4 in. to 6 ft 7% in. #4 bars at 10.4 in. #3 bars at 12 in.*

Analysis of calculations:
A fl includes v. reduction due to torsion.
Location from
beam end A,+ 2 A t Ar
0ftto4ft1%in. #4 barsat5.6in.
0ftto4ft7in. — #4 barsat12in.*
4 ft 1% in. to midspan #4 bars at 6.8 in.

*At each web face.

Additional reinforcement: 2. Furnish #4 bars at 12 in. center-to-


center plus two additional bars at
1. Provide two additional #4 stirrups at each tee stem load for ledge flexural
end V. tee stem loads. reinforcement.

128
EXAMPLE 2-PARKING GARAGE SPANDREL BEAM
LEDGE CORBEL

Refer to Fig. 27c for spandrel beam de- From Eq. (19):
tails. 0.08 (21) 12.5
Any «m,nfmY^^ = 60
Design Data
= 0.35 sq in.
1. See Example 1 for loads and dimen-
sions. Using Eq. (20), determine A1.5 steel:
2. The beam-to-column details are
shown in Fig. 27c. The spandrel _ 153.8
beam is torsionally stable without A "` 0.85 (60) 4 (1) 21 (12.5)
any end connections.
= 0.44 sq in. (controls)
3. Design corbel end using method
based on corbel behavior. From Eq. (21):
4. Refer to Fig. 25 for corbel require-
ments. 0.04 (21) 12.5
5. Consider ledge corbel loads to result A (minimum) 60
only from applied tee stem loads. = 0.18 sq in.

Determine Ultimate Applied Loads DistributeA 15 per Fig. 25c and consider
Ultimate shear load: only one leg (lower) of closed stirrup
effective (see Fig. 25d).
V. = (4/3) (1.52) (3) (25.3)
= 153.8 kips Spacing for #4 bars:

Ultimate shear stress: 1.5 b, (bar area) _ 21 (0.20) 9.6 in.


Estimate d, = 1.2.5 in. A n^ 0.44 -
Corbel width = 1.5 h = 1.5 (14) = 21 in.
Maximum spacing:
V. = 153,800 h/2 = 14/2 = 7 in. center-to-center
01= 586 psi
b, c1 21 (12.5) p
Using Eq. (22) determine A ft steel:
The calculated shear stress is therefore
Estimate d 2 = 10 - 1.5 = 8.5 in.
satisfactory since it is less than the
recommended 800 psi maximum. = 12.5
A,2 8 5 (1.09) = 1.60 sq in.
Find Ledge Corbel Reinforcement
Using Eq. (17), determine A 11 steel: Select #4 bars.
Total required = (1.60/0.20) = 8 bars
= 3 (153.8) 6 Over 1.5 h = 21 in.
= 1.09 sq in,
0.85 (4) 60 (12.5)
Note that no ledge attachment rein-
This steel area controls. Select four #5 forcement is required due to beam reac-
bars with welded cross-bars. tion for end tee stem V..
Using Eq. (18):
Add #4 stirrups so that additional stir-
_ (153.81 rups plus shear and torsion stirrups pro-
`^" 0.85(60)22(1)21(12.5) vide 1.60 sq in. over the beam's end 21
= 0.88 sq in. in.

PCI JOURNAUMarch-April 1984 129


Design Summary stresses at a working load of 3 (25.3)
1. Cross-harwelds (#5 cross-har) for the• = 75.9 kips is required considering
four #5 A 8 , bars should he in accor- the bearing is not uniform-
dance with the PCI Design Hand- 3. The ultimate concrete bearing stress
book requirements and completely on the extreme edge of the ledge may
specified. require the use of additional bearing
2. Analysis of bearing pad bearing reinforcement according to Fig. 39.

EXAMPLE 3- DETERMINATION OF PRINCIPLE MOMENTS


OF INERTIA FOR A BUILDING SPANDREL BEAM

Assume a spandrel beam for a build-


ing with a configuration and dimensions
shown in cross section below. Divide r
beam into two parts.
PARrO

Ail
Beam Properties
x 8.12 in.
y = 15.66 in.
I 54,024 in.4
Sn = 3450 in? ^
I,, = 13,283 in.4
A l = 476 sq in.
A E = 60 sq in. y_ PARtO
^ X
Determine Product of inertial,„ Building beam data.
From strength of materials:

IS„= 1xtytAi
Isv = x , y , A, + xa y z As
x, =7-8.12=-1.12 in.
y, = 17- 15.66= +1.34 in.
xz = 17 – 8.12 = +8.88 in. 'VfX
yZ = 5– 15.66= –10.66 in.
C.G•Q
Izj = (-1.12) (+ 1.34) 476 + (+8.88) x
( 10.66) 60 = – 6394 in.'

Find Orientation Angle t


X
Refer to Fig. 26 and use Eq. (23):

B= tan' [ 2 (-6394)
2 13,283 – 54,024
_ + 8.71 deg (counterclockwise)
Determination of principle moments of
Find 'max Inertia.
Use Eq. (25):

130
, max — 54,024 + 13,283 +
2
( 54,024 — 13,283 )2 + 1P
( - 6394)
l 2 J
= 55,003 in

Find Imin
c,
L ___7Lp
Use Eq. (26): ^Q X `4
54,024 + 13,283 1IbPX
I ^rn = 2
( 54,024 — 13,283 ^^ +(_6394)2
2 yi
12,304 i.n,4
ly
P
Find Sb min Associated With 'max Determination of minimum section
a = , (15.66) 2 + (11.88)2 modulus.
= 19.66 in,
11.88 from the beam center of gravity to the
^3 =sin 1
19.66 strand center of gravity where the
= 37.18 deg distance is parallel to line y, — yF.
2. The above procedure for unsymmet-
(3-8=37.18-8.71 rical members can be used generally
= 28. 47 deg for all precast prestressed concrete
members.
Y bn = (a) cos (ji — B) 3. A graphical approach is recom-
= 19.66 cos (28.47) mended for determining all dimen-
= 17.28 in. sions such as ya p once B, I m . r , and
I,,,,, have been calculated.
Determine S b mi at Point A
Imar
b mth = METRIC (SI) CONVERSION
FACTORS
_ 55,003
1 ft = 0.305 ft
17.28 1 sq ft = 0.0929 m2
= 3188 in," t in. =25.4mm
1 sq in. = 645.2 mm2
which is less than the value ofS b = 3450 I in.3 = 16,388 mm3
in.3 for I , found previously.
1 in.4 = 42,077 him;
L kip = 4.448 kN
Summary I lb = 4.448 N
1. Ifthe beam is prestressed, the inter- 1 ksi = 6.895 MPa
nal prestress moment is the product 1 psi = 0.006895 MPa
of the strand force times the distance 1 psf = 0.04788 kPa

PCI JOURNAllMarch-April 1984 131

Das könnte Ihnen auch gefallen